You are on page 1of 21

1. fejezet.

A szociolgia mint tudomny


Struktra: Egy trsadalom a szociolgusok tbbsge szerint nem egyszeren a trsadalom tagjainak sszessge, mert van bizonyos struktrja (szerkezete). Trsadalmi struktrn a legltalnosabb rtelemben az egynek s trsadalmi csoportok kztti viszonylag tarts viszonyokat (pl.al- s flrendeltsget) rtjk. Intzmny: A trsadalom tagjainak tbbsge ltal vgzett tevkenysgek alapvet mdja,mintja. Az intzmnyekhez normk s rtkek tartoznak, amelyek az intzmnyeslt viselkedst elrjk, tovbb megszegst bntetik. Kultra: Az anyagi javak,normk s rtkek egyttese. Homo oeconomicus: A klasszikus kzgazdasgtan emberkpe. Miszerint az ember mindig racionlisan dnt, cselekszik, tovbb a haszna maximalizlsra trekszik, azaz sszehasonltja az adott cl elrshez szksges rfordtsokat s az ltala elrhet hasznokat, s gy dnt, hogy az adott rfordtssal maximlis hasznot rjen el, vagy megfordtva- az adott hasznot a lehet legkisebb rfordtssal rje el. Homo sociologicus: A szociolgia emberkpe, miszerint az ember tevkenysgben a trsadalmi normkhoz igyekszik alkalmazkodni. E normk htterben rtkek llnak. A normkat s rtkeket a trsadalom tagjai gyermekkori szocializcijuk sokn sajttjk el. Ideolgia: Olyan vilgkp, amely az emberek egy rsznek rdekeit szolglja VITAKRDSEK 1. Megismerhet-e az igazsg a trsadalom tudomnyos kutatsa tjn? A szociolgusok tbbsge azt vallja, hogy ha soha nem is lehetnk biztosak abban, hogy megtalltuk az igazsgot, kutatsaink kzelebb visznek a megismershez, azaz minden j hipotzissel, amelyet a megcfoldott korbbi hipotzis helyett fogalmazunk meg, kzelebb kerlnk az igazsghoz. Tudomnyfilozfiai vitakrds, hogy a tudomnyok kzttk a szociolgia gy fejldnek-e, hogy a korbbi ismeretekre ptve egyre tbb j ismeretet halmozunk fel, vagy pedig tudomnyos forradalmak tjn, vagyis gy, hogy idvel felismerjk, hogy halmozd ismereteink nem magyarzzk meg a valsgot, ezrt flredobjuk az addigi megkzeltst, az j krdseket fogalmazunk meg, azaz j nzpontbl kzeltnk az igazsghoz. 2. A homo oeconomicus vagy a homo sociologicus, vagy esetleg valamely ms tudomny emberkpe ll-e kzelebb a valsghoz? A szociolgia a tbbi trsadalomtudomnytl emberkpe tekintetben is klnbzik. Kzismert a klasszikus gazdasgtan emberkpe, a homo oeconomicus. Eszerint az ember mindig racionlisan dnt, a haszna maximalizlsra trekszik. A szociolgia viszont a homo sociologicus (Dahrendorf 1958, Andorka 1995) emberkpbl indul ki. Eszerint az ember tevkenysgben a trsadalmi normkhoz igyekszik alkalmazkodni. E normk htterben rtkek llnak. A pszicholgiban tbb emberkppel is tallkozhatunk, de valamennyi irnyzat slyt helyez az egyni lelki tnyezk szerepre, pl. a pszichoanalzis szerint az egynt jelents mrtkben tudat alatti motvumok irnytjk, s ezek ersebbek a haszonmaximalizlsnl s a normakvetsnl. Ennek alapjn beszlhetnk homo psychologicusrl is. A politkatudomny tbbnyire abbl indul ki, hogy az embert alapveten a hatalomvgy, a hatalom megszerzse s megtartsa mozgatja. Ezt az emberkpet homo politicusnak nevezhetjk. 3. Mire hasznlhat s mire nem a szociolgia? A trsadalmi nismeret nvelsre, konkrt problmk megoldsra, elrejelzsre, tervezsre, a jvbeli trsadalmi fejlds kimutatsra? Nyilvnvalan a szociolgiban jrtas egyn jobban megrti a trsadalmat, amelyben l, ezrt jobban tud vdekezni minden manipulcival, demaggival, flrevezet nzettel szemben. Konkrt trsadalmi problmk megoldst segti el azzal, hogy ismereteket ad ezeknek a problmknak a megismershez. Felhasznlhat az elrejelzsben s tervezsben az igazi tudomny kritriuma egyes vlemnyek szerint, hogy elre jelezni tudja az esemnyeket, folyamatokat. Nagy trtneti fordulatokat, gyors s radiklis vltozsokat a szociolgia ltalban nem tud elre jelezni, megjsolni. Trsadalmi tervezs pl. egy telepls fejldst lehet lehet politikai eszkzkkel befolysolni, azonban illzi, hogy a trsadalom egsze kzpontilag tervezhet. A szociolgia nem alkalmas arra, hogy a gazdasgi s trsadalmi fejlds nagy vszzados irnyait, vltozsait megjsolja. Robert K. Merton (1980) amerikai szociolgis szerint a szociolgia kzpszint trvnyszersgek kutatsra hasznlhat, azaz legfeljebb kzpszint elmleteket fogalmazhat meg. Kzpszint elmlet az olyan elmlet, mely egy jl definilhat jelensg (pl. devins viselkedsek) elfordulst rtelmezi egy adott trsadalomban s korszakban. 4. Hogyan viszonyuljon a szociolgus a politikhoz?

Minden szociolgusnl felvetdik a szociolgia tudomnya s a politikai cselekvs kztti viszony krdse. Max Weber vizsglta a legmlyebben a tudsi s a politikai tevkenysg kzti klnbsgeket. A tudsnak szerinte arra kell trekednie, hogy megllaptsait, ismereteit, mdszereit szemlletes pldja szerint a hv katolikus s a szabadkmves egyformn felhasznlhassa. A tuds szmra a legfbb rtk, hogy a tudomnyos igazsgot kimondja, legalbbis kzeledjen afel. a politkus ezzel szemben arra trekszik, hogy hatalomra jusson, a hatalmat megtartsa, vagy Max Weber fennkltebb megfogalmazsban politikai cljait, eszmnyeit a trsadalomban megvalstsa. A szociolgusnak gyelni kell arra, hogy vlemnye megformlsban ne a kormnyzat ignyeit, hanem sajt tudomnyos meggyzdseit kvesse. 5. Lehet-e a szociolgus trgyilagos, amikor a trsadalmi jelensgeket kutatja? Melyek az objektivits felttelei? A tudomnynak objektv megllaptsokat kell megfogalmaznia, s a tudomnyt nem befolysolhatjk a kormny, a prtok, a klnbz trsadalmi csoportok, de mg a kutat sajt politikai nzetei, rdekei sem. Ngy fontos kvetkeztets megfogalmazhat: A szociolgusnak tisztban kell lennie azzal, hogy minden trsadalomtuds munkssgt, gy az vt is szksgkppen befolysolja a sajt trsadalmi helyzete, politikai llsfoglalsa. A szociolgusnak arra kell trekednie, hogy kutatsait minl kevsb befolysoljk a sajt rtkei, minl inkbb az objektv igazsg, a valsg feltrst szolglja. A trsadalmi valsg trgyilagos szociolgiai feltrsa szksgszeren elsegti a trsadalmi viszonyok javulst. A szociolgus az eredmnyek kzzttelekor s elemzsekor semmikppen sem hagyhatja figyelmen kvl azt, hogy tudomnyos megllaptsait milyen clra, milyen gyek altmasztsra lehet felhasznlni. Trekednie kell arra, hogy a kutats eredmnyeit trgyilagosan mutassa be.

2. fejezet. A szocilgia trtnete


ipari trsadalom: Els megfogalmazja Claude Henri Saint-Simon (1760-1825). Az ipari trsadalomban az emberek kormnyzst fel fogja vltani a dolgok adminisztrcija. A gazdasgi s trsadalmi folyamatokat tervezni fogjk. Az ipari trsadalomban a vezet osztly szerinte tudsokbl, bankrokbl, ipari vllalkozkbl, mrnkkbl fog llni. Kapitalista rendszer: A kapitalista trsadalom alapvet jellemzje, hogy a tksosztly ll a munksosztllyal szemben, a tksek kizskmnyoljk a munksokat. A munksok szemlyileg szabadok, de meglhetsk rdekben knytelenek brmunksknt eladni munkjukat a tkseknek, akik a termeleszkzk birtokban vannak. Szocialista rendszer: A szocializmusban a jvedelmeket a vgzett munka arnyban osztjk el. A szocialista rendszer msik f jellemzje a termeleszkzk trsadalmi tulajdonba vtelben lttk. Formcielmlet: Egy nagy trtnelmi fejldsi elmlet, miszerint az emberisg az skzssgi trsadalom utn a rabszolgatart, a feudlis, a kapitalista s a szocialista rendszeren megy keresztl, mg vgl a fejlds a kommunista trsadalomban cscsosodik ki. A rabszolgatart trsadalomtl a kapitalista trsadalomig a kizskmnyols jellemezte a rendszert. Konfliktuselmlet: Marx ttele, hogy a konfliktus az emberi trsadalmak lnyeghez tartozik, st a konfliktus a trsadalmak fejldsnek egyik mozgatereje. A mai szociolgia konfliktuselmleti irnyzata azt lltja, hogy minden emberi trsadalomban voltak ,vannak s lesznek konfliktusok. Ezen irnyzat szerint a konfliktus pozitv szerepet jtszik, ha a trsadalom intzmnyei azt megfelelen tudjk kezelni, de slyosan veszlyeztetheti is a trsadalom integrcijt s ezltal fennmaradst is,ha kezelhetetlenn vlik. Harmoniaelmlet: A szociolgia konfliktuselmleti irnyzatval szemben ll a funkcionalista irnyzat, vagy harmniaelmlet, amely a trsadalmak lnyegt az egyttmkdsben ltja, s a konfliktusban trsadalmi problmt lt. Ez az irnyzat igen kritikus Marxszal szemben. Brokrcia: A racionalizlds megnyilvnulsa a gazdasgi,trsadalmi s politikai szervezetekben a brokratizlds. Webernl a brokrcia azt jelenti,hogy a szervezetek az gyeket pontosan meghatrozott szablyok szerint intzik. A brokrats kztt pontos munkamegoszts rvnyesl, szakismereteik, pontosan megfogalmazott kvetelmnyek alapjn vlasztjk ki ket. A brokrcia mkdse teht kiszmthat. Fenomenolgiai szociolgia: Alfred Schtz dolgozta ki, lnyege,hogy a szociolgiai kutats figyelmnek az emberek mindennapi letre kell irnyulnia, azaz hogyan rtelmezik az emberek az ket krlvev trsadalmi krnyezetet, hogy igazodnak el a trsadalomban. Funckionalizmus: Szoks f harmniaelmletnek nevezni. Talcott Parsons (1902-1979) vezette be a szociolgiba. Abbl indul ki,hogy minden trsadalomnak 4 feladatot kell elltnia, azaz 4 funkcija van, s ennek megfelelen minden trsadalmon mint rendszeren bell ngy tovbbi alrendszer mkdik: Manifeszt funkci: (Robert Merton ) Manifeszt= nylt ( pl nagy s gyors kocsit azrt vesznek az emberek, hogy biztonsgosan s gyorsan kzlekedjenek)

Latens funci: = rejtett funkci elbbi kocsis pldnl, hogy a vsrl lthatv tegye magas trsadalmi sttust. Strukturalizmus: A nyelvtudomnyban jelent meg, a nyelvekben megtallhat struktrk kutatsra helyezte a slyt. Claude LviStrauss francia etnolgus vezette be a trsadalomtudomnyokba. Szimbolikus interakcionizmus: Mead elmletre tmaszkodik. A trsadalmat az emberek kztti kapcsolatok, interakcik megfigyelsn keresztl kvnja megismerni. A makroszociolgiai krdsek helyett a mikroszociolgiai jelensgekre, a mindennapi letre, az emberek kztti kapcsolatokra sszpontostja a figyelmt. VITAKRDSEK 1. Marx, Durkheim vagy Weber gondolataibl melyeket lehet a mai trsadalom vizsglatban felhasznlni? Karl Marx (1818-1883) vezette be a szociolgiban ma is hasznlt fogalmakat s gondolatokat: a trsadalmi-gazdasgi formcik, kzttk klnsen a feudalizmus s a kapitalizmus,a trsadalmi osztlyok, a konfliktusok, mint a trsadalom lnyeghez tartoz jelensgek, az elidegeneds, a gazdasgi alap s felptmny viszonya. Ersen hat ma is a szociolgira Marx azon ttele, hogy a konfliktus az emberi trsadalmak lnyeghez tartozik, st a konfliktus a trsadalmak fejldsnek egyik mozgatereje. Marx szerint a szocialista trsadalomban eltnik a konfliktus, a kommunizmusban teljes lesz a trsadalmi harmnia. Emile Durkheim (18581917) a trsadalmi integrci feltteleit kereste. Az anmia fogalmt Durkheim dolgozta ki s vezette be a szociolgiba. Ha a normkban val egyetrts meggyengl, megn a klnfle trsadalmi problmk gyakorisga. A trsadalmon belli konszenzus s szolidarits megteremtsben nagy jelentsget tulajdont a vallsnak.Fontos ttele, hogy a trsadalmi tnyeket csak ms trsadalmi tnyekkel lehet megmagyarzni. Az ngyilkossg ugyan egyni cselekedet, de az ngyilkossg gyakorisga trsadalmi tny, ezrt nem magyarzhat az egynek tulajdonsgaival, hanem csakis trsadalmi tnyekkel. Max Weber (1864-1920) Marx elmleteire, gondolataira reaglt. Szerinte a trsadalmi szerkezet differencildik a gazdasg, a hatalom, az letmd s a megbecsltsg dimenziban. Weber a hatalomnak, s ezltal a politiknak sokkal tbb figyelmet szentelt, mint Marx. Megklnbztette a hatalmat, mely a fizikai knyszer fenyegetsn alapul, s az uralmat, A racionalizlds megnyilvnulsa a gazdasgi, trsadalmi s politikai szervezetekben a brokratizlds. Kortsaival ellenttben Weber szerint a trsadalomban s a trtnelemben nem determinisztikus trvnyek, hanem valsznsgi jelleg sszefggsek rvnyeslnek. 2. A Parsons-fle funkcionalizmus s a nyugati marxistk vitjban kiknek miben volt igazuk? A funkcionalizmust Talcott Parsons (1902-1979) vezette be a szociolgiba. Abbl indul ki, hogy minden trsadalomnak ngy feladatot kell elltnia, azaz ngy funkcija van, s ennek megfelelen minden trsadalom mint rendszeren bell tovbbi alrendszer mkdik: politika a trsadalom cljainak meghatrozsa gazdasg az eszkzk hozzrendelse a clokhoz, vagyis a clok megvalstsa. kultra a trsadalom tagjainak, tevkenysgknek integrlsa szemlyisg a trsadalom fennmaradshoz szksges rtkek s normk tadsa az egyik nemzedktl a msiknak, szocializci A funkcionalizmus msik kiemelked amerikai kpviselje, Robert Merton (1910-) nem fogalmazott meg Parsonhoz hasonl elmletet, viszont tbb kritikus ponton mdostotta s kiegsztette a funkcionalizmust, gy elfogadhatbb tette azt a szociolgusok tbbsge szmra: - rmutatott, hogy egy-egy fennll intzmnynek, normnak, hiedelemnek nemcsak funkcii, hanem diszfunkcii (htrnyos hatsai) is lehetnek. - megklnbztette a manifeszt (nylt) s a latens (rejtett) funkcikat, ezltal lehetv tette, hogy a szociolgiai elemzs a jelensgek felsznnl mlyebbre hatoljon. - ugyanaz az intzmny, norma, hiedelem funkcionlis lehet a trsadalom egyik osztlya s diszfunkcionlis a msik osztlya szmra. Marxista irnyzatok - a korn elhunyt C. Wright Mills (19161962) az amerikai szociolgia fenegyerekevolt. Brlta Parsons fogalmi s Lazersfeld mdszertani fetisizmust, velk ellenttben a szociolgiai fantzia szksgessgt hangoztatta. Kzel llt a marxizmushoz a Frankfurti Iskola, amely Frankfurtban jtt ltre az 1920-as vekben. Az iskolhoz tartoz tudsok: Max Horkheimer (18951973), Theodor W. Andorn (1903-1969) tovbb a msodik s harmadik nemzedkbl Jrgen Habermas. A Frankfurti Iskola tagjai az antiszemitizmus gykereit a szemlyisgben, az antiszemitizmusra hajlamos szemlyisg kialakulsnak okt pedig a gyermekkorban kapott nevelsben kerestk, mutatja a pszichoanalzis hatst a gondolkodsunkra. A Frankfurti Iskola ma is l s publikl kpviselje Jrgen Habermas (1929- ). Inkbb filozfus, mint elmleti szociolgus. Habermas vitatkozott egyrszt a pozitivista szociolgusokkal (Papp 1976), msrszt a funkcionalistkkal, klnsen Luhmann-nal. Knyvet rt a trtnelmi materializmus rekonstrukcijrl, ebben messze eltvolodott a marxizmus eredeti tantsaitl. 3. Szksg van-e kzssgekre a mai trsadalmak j mkdshez? James Coleman egyik tanulmnyban azt hangslyozta, hogy a modern trsadalmakban egyre nagyobb szerepet jtszanak az ember alkotta trsadalmi szervezetek, s az ezekben felmerl elosztsi s kontroll problmk kezelshez jfajta kzssgek s szablyok szksgesek. A szlssges individualizmus reakcijaknt jelent meg a kommunitarizmus, vagyis a kzssgiessg irnyzata. (Walzer 1980, Etzioni 1988, Bellah et al.1991, Honneth 1993) Ez az elmlet 2 ponton ellentmond az individualizmusnak: - az egyes emberek jlte s a trsadalom mkdse rdekben szksg van kzssgekre - tovbb az erklcs s azt altmaszt rtkek nlkl a modern gazdasg s a politikai demokrcia nem kpes hatkonyan mkdni 4. Szksg van-e erklcsre s rtkekre a modern gazdasg s a demokrcia mkdshez? Lehet-e trsadalmi egyetrtst teremteni az erklcsi normkat s rtkeket illeten?

Nhny vtizeddel ezeltt meglehetsen ltalnos volt az a felfogs, hogy szksges az egyetrts a modern trsadalomban a normkat s rtkeket illeten. Mind Durkheim, mind Parsons s t kveten a funkcionalistk gy gondoltk, hogy a trsadalom mkdse szinte elkpzelhetetlen a kzs normk s a mgttk ll rtkek hinyban. Ha egy adott szemly megszegi ezeket a normkat, akkor devinsnak minsl, ha pedig az egsz trsadalomban meggyenglnek a normk s az rtkek, akkor bekvetkezik az anmia llapota, amely a trsadalom egszt veszlyezteti. Mg a Frankfurti Iskola s a harmadik nemzedkhez tartozk is ez egsz rendszerre nzve veszlyesnek lttk, ha az alapelveket illet konszenzus megsznik, mert ezltal a legitimci vsz el. A mai nyugati trsadalmak azonban, klnskppen az amerikai, egyre inkbb multikulturliss vltak, vagyis egyre nagyobb szm, a tbbsgtl eltr nemzeti s vallsi kultrj ember l bennk. Ez a tny az alapja annak a liberlis trsadalomfilozfusok egy rsze ltal vallott felfogsnak, hogy nem lehet s nem szabad norma- s rtkkonszenzusra trekedni, ezeknek hinyban kell a modern trsadalom intzmnyeit alaktani gy,hogy azok mkdkpesek legyenek. John Rawls Politikai Liberalizmus c. mvben azt mondja, hogy az rtkekben nem lehet megegyezni, csak demokratikus eljrsi szablyokban. Aki ezeket elfogadja, annak eltr rtkeit s normit a trsadalomnak tolerlnia kell. Msok, akik a kommunitrius irnyzatokhoz llnak kzelebb, vltozatlanul hangslyozzk, hogy a kzs rtkek s ltalnos elfogadott viselkedsi normk nlkl sem a gazdasg, sem a trsadalom, sem pedig a demokratikus politikai intzmnyek nem mkdkpesek.

3 . fejezet Egyenltlensg, szegnysg


Egyenlsg: megklnbztethet a pozicik egyenlsge s a pozicikba val bejuts eslyeinek egyenlsge. A pozicik egyenlsge azt jelenti hogy a trs. minden tagjnak, foglalkozstl, iskolai vgzettsgtl stb. fggetlenl, egyenl a jvedelme. Tgabb rtelemben az egyenlsg kiterjed a laksviszonyokra, vagyonra, lakhelyre, st az elgedettsgre is. A pozicikba val bejuts eslyeinek egyenlsge azt jelenti, hogy a trsadalom minden tagjnak egyenl eslye van arra, hogy a magasabb jvedelmet s ms privilgiumokat biztosit pozicikba bejusson. Mltnyossg: mltnyosnak nevezzk a trsadalmi berendezkedst, a jvedelemeloszlst stb, ha minden llampolgr akkora javadalmazsban rszesl, amelyik arnyos a trsadalomnak nyjtott szolglatval. ( a trs., ahol ez megvalsul a meritokrcia.) Igazsgossg: igazsgosnak nevezik azt a trsadalmat vagy jvedelemeloszlst, amelyet a trsadalom minden tagja elfogadhatnak tartana egy olyan kiindul helyzetben, amikor mg nem tudja, hogy kedvezbb vagy kedveztlenebb pozicit fog elfoglalni a trsadalomban. Szegnysg: szegnynek mondjuk egy trsadalmon bell azokat, akiknek jvedelme a kivlasztott szegnysgi kszbnl alacsonyabb. A szegnysgi kszb lehet egy valamilyen mdszerrel meghatrozott ltminimum, ebben az esetben beszlnk abszolt szegnysgrl. De a szegnysgi kszbt meghatrozhatjuk az adott trsadalomban tlagos jvedelem valamilyen szzalkban, ebben az esetben beszlhetnk relativ szegnysgrl. Vgl szegnynek mondhatjuk a trsadalom legalacsonyabb jvedelm rszt, pld. 10 vagy 20 szzalkt. A szegnysgfogalmat ki lehet terjeszteni olyan mdon, hogy nemcsak az alacsony jvedelmeket tekintjk szegnyeknek, hanem azokat is, akik ms ok miatt (betegsg, alacsony iskolai vgzettsg) vannak az tlagosnl lnyegesen rosszabb helyzetben. Az utbbi tipus szegnysget szoktk deprivcinak (valamitl val megfosztottsgnak) is nevezni. Ltminimum: az az sszeg, mely egy embernek szksges ahhoz, hogy alapvet szksgleteit minimlis szinten ki tudja elgiteni. Legtbbszr gy szmitjk ki, hogy sszellitanak egy, a tpanyagszksgletet fedez lehet legolcsbb trendet, s ennek rhoz hozzadjk az egyb szksgletek fedezsnek szzalkosan becslt ktg-t. Szegnysgi kszb: az a jvedelemszint, amelynl kisebb jvedelmeket szegnynek tekintjk. A szegnysgi kszb lehet a ltminimum, de meghatrozhat ms mdon is, pld az tlagjvedelem bizonyos szzalkban. Deciliseloszls: a szociolgiai adatfelvtel sorn megfigyelt rtkek 10 egyforma esetszm kategriba rendezse a megfigyelt rtkek nagysga szerint. A legals decilisbe kerl pld a megfigyelteknek az a 10 szzalka, akinl a legalacsonyabb rtket figyeltk meg, a msodik decilisbe a kv legalacsonyabb 10 szzalk stb. GDP- gross domestic product brutt hazai termk. Az adott vben belfldn elllitott termkek s szolgltatsok sszege, pontosabban a hozzadott rtkek (a kibocsts s rfordits klnbsge) sszege, az lleszkzk amortizcijnak levonsa nlkl. A nemzetgazdasg vi teljesitmnyt mri. Az egy fre jut GDP-t nem lehet egyrtelmen a jlt mutatjnak tekinteni, mert a jltet szmos olyan pozitiv s negativ tnyez (krnyezetszennyezs stb) is befolysolja, amelyet a GDP-ben nem vesznek szmitsba. Munkanlklisgi arny: mskpp munkanlklisgi rta. A regisztrlt munkanlkliek szma osztva a gazdasgilag aktiv npessg (foglalkoztatottak s munkanlkliek sszege) szmval. Regisztrlt munkanlkli az, akinek nincs munkaviszonya, nem nyugdijas, nem tanul s munkt keres. Jlti llam: -nak nevezzk azoknak a jlti programoknak az egyttest, amelyeken keresztl az llam messzemenen gondoskodik polgrainak jltrl. A jlti llamok a II. VH. Utn fejldtek ki Nyugat-Eu-ban. Az ltaluk nyjtott tmogatsok mretei s konkrt tipusai is meglehetsen eltrek. Jlti programok: ide soroljuk a nyugdijat, a tppnzt, a balesetbiztositst, a munkanlklibiztositst, a csaldi tmogatsokat(cs.ptlk, anyasgi seglyek stb) az ingyenes eggelltst, s nmelyek az ingyenes oktatst is. 1. A szegnysg okai ltalban a vilgon, a fejlett orszgokban s specilisan a magyar trsadalomban. ltalnos okok: Szlssges llspontok: Biolgiai adottsgok hatrozzk meg az intelligencia szintet, rkldik.(nem bizonytott), A gazdasgi-trsadalmi rendszer jellege s mkdse, azt kell megvltoztatni. Egyb llspontok: Mveltsg s iskolai vgzettsg tern fennll htrnyok. - oktats Rossz testi s lelki egyszsgi llapot a krnikus betegek, rokkantak helyzete A csonka (egyszls) csaldok vlnak szegnny. k mintjukat trktik

Nagyjbl a fenti 3 elmlet tvzse a szegnysg kultrja elmlet: A szegny rtegbe tartozknak egy egszen klns viselkedsi norma s rtkrendszere, vagyis kultrja van. Ez megakadlyozza ket abban, hogy a szegnysgbl kiemelkedjenek, viszont az ezzel jr terheket knnyebben elviselik. letciklus-jelensg: Klnfle demogrfiai esemnyek (hzassgkts, zvegyls, vls, az tlagosnl tbb gyermek szletse) A fejlett orszgokban a szegnysg: kzvetlen oka: br az egy fre jut jvedelem orszgos tlaga emelkedett, a jvedelemegyenltlensg ntt (Nmetorszgban is),a trsadalom legalacsonyabb szint kategrijnak helyzete romlott. kzvetett oka: a kormnyok arra trekedtek, hogy adpolitikjukkal s a trsadalmi juttatsok megnyirblsval tformljk a jvedelem-eloszlst. Tovbbi okok: Munkanlklisg, fknt a tarts munkanlklisg Amerikban a gyermekek krben is egyre meghatrozbb a szegnysg, ezzel szemben az idsebb emberek kztt ltalnossgban cskkent a szegnysg arnya Egyszls csaldok Az egyedl l ids nk kztt gyakoribb a szegnysg Etnikai kisebbsg, bevndorlk, segdmunksok Demogrfiai esemnyek (gyermek szletse, nyugdjkorhatr elrse, zvegyls, vls) Magyarorszgon: A szocialista korszakban: a relbrek 1978-ig nttek, azutn cskkentek. A rendszervlts ta: 1990-tl a jvedelemegyenltlensgek lnyeges emelkedse Oka: a GDP cskkense, a munkanlklisg, az inflci 2. Milyen jvedelem-egyenltlensgeket tartunk kvnatosnak vagy elfogadhatnak a mai Magyarorszgon? Kvnatos lenne a skandinv szint mint a szocializmusban volt. 95-ben akkora volt mint Nyugat-nmetorszgban s nem szeretnnk elrni az USA, vagy plne a latin-amerikai szintet. Az elfogadhat egyetltlensgek mrtke elmletek. (3) 1. Az egyenltlensgek szksgszerek, kvnatosak Elitista llspont: az elit az tlagnl tehetsgesebb, intelligensebb, stb., ezrt magasabb jvedelmeket rdemelnek. Libertarinus llspont: minden llami beavatkozs a szegnyek rdekben, srti a gazdagok szabadsgjogait s diktatrhoz vezet. Funkcionalista szociolgiai iskola: a fennll jvedelemeloszls azt tkrzi, hogy a trsadalom tagjai mennyire hasznos funkcit tltenek be a trsadalom szmra. 2. A mrskelt egyenltlensg elfogadhat, de az eslyek legyenek egyenlek Akik erre az llspontra helyezkednek, azokat Amerikban liberlisnak, Eurpban szocildemokratknak nevezik. Az igazsgos trsadalom kt f elve: a trsadalom minden tagjnak egyenl joga van az alapvet szabadsgjogokhoz, addig a hatrig, mg az sszeegyeztethet a trs tbbi tagjnak hasonl teljes kr szabadsghoz. a trsadalmi egyenltlensgeket gy kell elrendezni, hogy azok hosszabb tvon a legszegnyebbek helyzett is javtsk, s a trs minden tagjnak egyenl eslye legyen a kedvez pozicikba jutshoz. 3. Teljes egyenlsg = egalitrius llspont Marx s Engels szerint a kommunista trsadalomban a teljes egyenlsg megvalsul. A szocializmusban ezt sem lehetsgesnek, sem kvnatosnak nem tartottk, ehelyett a munka szerinti elosztst hirdettk. A mai magyar trsadalomban kvnatos cl: 1.a trsadalmi egyenltlensgek minden olyan cskkentse, ami a gazdasgi fejldst nem htrltatja s nem korltozza elfogadhatatlanul az egyni szabadsgot. 2. Senkinek se kelljen lnie a ltminimumnl kisebb jvedelembl, s senki se szenvedjen egyb tnyezk miatt slyos htrnyokat. 3. Melyek az egyes klnsen veszlyeztetett kategrik a romk, a gyerekek, a munkanlkliek szegnysgnek okai, s milyen eszkzkkel lehetne szegnysgket enyhteni? A szegnysg okai: Romk: alacsony iskolai vgzettsg; azonos kpzettsg mellett is kisebb az eslyk a foglalkoztatsra; alacsonyabb sttus munkt kapnak; magas gyermekszm Gyermekek: cskkent a trsadalmi jvedelmek (pl. csaldi tmogatsok) relrtke; nincs keresetk; Munkanlkliek: nincs rendszeres keresetet biztost munkahelyk, vagy nem rszeslnek elegend nyugdjban; szerkezettalakuls, gyrbezrsok, racionalizls a rendszervlts utn sok munkanlkli, de nehz volt munkhoz jutni; a jvedelmi egyenltlensgek dinamikusan nvekedetek, s a jvedelmi pozcik trendezdtek. Megolds: gazdasgi fejlds, jlti rendszere 4. Milyen fok s milyen jelleg reform szksges a magyarorszgi jlti rendszerben? A jlti rendszer reformja, nem pedig a leptse szksges, egy, a magyarorszgi lehetsgeknek megfelel korszer jlti rendszert kellene felpteni. Minden egyes jlti tmogatsfajta reformjt kln-kln kell megvizsglni, figyelembe vve, hogy a reform kiket rinthet htrnyosan, milyen messzemen hatsai lehetnek, s a vrt megtakartsok mellett milyen tbbletkltsgeket okozhat (pld jvedelemigazolsok ellenrzse) Kvnatos lenne, ha az llami szocilpolitika mellett nem llami szervezetek, intzmnyek (egyhzak, egyesletek) is minl nagyobb rszt vllalnnak a szegnysg enyhtsben. Vgl a csaldok fontossga miatt szksges lenne a csaldokat segteni. 5. Melyek a szegnysg genercis jellegzetessgei Eurpban s Magyarorszgon?

Volt szocialista orszgok: gyerekek htrnyos, idsek relatve elnys helyzetben Csehorszgban, Lengyelorszgban s Magyarorszgon a 18 v alattiak szegnysgarnya az tlag msflszeresnl is magasabb, de nhny nyugat-eurpai orszgban (Ausztria, Hollandi, Anglia) is tlag feletti Belgium, s nmely szaki orszg: ppen fordtva a gyerekek jlti helyzete az ltalnosnl kedvezbb, az idsek rosszabb. Tbbi eurpai orszg: fiatalok s idsek szegnysgkockzata kztt nincs jelents klnbsg 6. Milyen idszakokat tudunk megklnbztetni az egyenltlensgek alakulsban Magyarorszgon a rendszervltst kveten? Tth Istvn szerint az egyenltlensgek alakulsa haznkban a rendszervlts utn: 1. szakasz (az 1890-es vek eleje): gazdasgi recesszi, szerkezettalakts jvedelmi egyenltlensgek dinamikus nvekedse, a jvedelmi pozcik trendezdse. Gyrbezrsok, 1991-92-ben hnapok alatt 600.000 munkanlkli. Munkajvedelmket egy alacsonyabb munkanlkli segly vltotta fel. 2. szakasz (az 1890-es vek kzepe): az egyenltlensgek (ugyan kisebb mrtkben) tovbb nttek. Ennek kt oka: A talpon maradt sikeres piaci szereplk jvedelemtbbletre tettek szert, gy mg jobban elhatroldtak a munkanlkliektl, mg nagyobb differenciltsgot kialaktva. Cskkent az egyes trsadalmi jvedelmek (munkanlkli segly, csaldi tmogatsok) relrtke fokozd gyermekszegnysg. 3. szakasz (az 1890-es vek vge): a jvedelmi pozcik stabilizldtak 7. Hogyan alaktjk az egyes trsadalmi tnyezk a szegnysgbe kerls kockzatait? Akik nagyobb valsznsggel lesznek szegnyek: Alacsony iskolai vgzettsgek (rtelemszeren: a magas iskolai vgzettsgek kisebb valsznsggel lesznek szegnyek, s gy tovbb) A betantott s segdmunksok A munkapiacrl kiszorulk (a munkanlkliek s az els llsukat keresk, a hztartsbeliek s az egyb inaktvak) A roma etnikumhoz tartozk Bizonyos demogrfiai jegyeket hordozk (sokgyerekes csaldok tagjai, a gyermekket egyedl nevelk, zvegyek) A htrnyos kisrgikban s teleplseken lak 8. Miben klnbzik a szegnysg statikus s dinamikus rtelmezse? A szociolgiai s statisztikai adatgyjtsek dnt tbbsge keresztmetszeti szemllet, ez pedig azt a benyomst alakthatja bennnk, hogy mind az elnys, mind pedig a htrnyos (szegnysgi) helyzetek statikusak, tartsak, alig alakthatak. A kvetses vizsglatok azonban kimutattk, hogy az egynek s csaldok pozcija nem felttlenl konzervldik. A modern trsadalom egyik alapjellemzje ppen az egynek trsadalmi pozcik kzti cserldse, az a jellegzetessg, hogy a trsadalom nem kasztosodik. Ennek alapja a modern trsadalmak dinamizmusa, a folytonos gazdasgi s trsadalmi innovci s verseny, a gazdasg hullmzsa. Ez a jellemz Magyarorszgon is tapasztalhat volt a rendszervlts utn: a jvedelmi pozcik kztt intenzv cserlds volt megfigyelhet, teht a 90-es vekben egyszerre volt jelen az elszegnyeds s a szegnysgbl val kilps. St a szegnysg ma sem eleve elrendelt helyzet: szegnny vlhatunk, de a szegnysgbl ki is lehet emelkedni. Ami ezeket a vltozsokat elidzi: a munkapiac kzvettette gazdasgi dinamika (foglalkoztatottbl munkanlkli, majd abbl jra foglalkoztatott); s az egyn lettjnak demogrfiai esemnyei (gyermekvllals, vrs, zveggy vls). 9. Milyen vltozsok kvetkeztek be a 90-es vekben Magyarorszgon a szocilis ellts terletn? Mrskldtt a jlti jraeloszts terjedelme: 1991 s 2001 kztt 5 szzalkponttal mrskldtt a szocilis jelleg kszpnzes kiadsok arnya. Nttek a keresetarnyos trsadalombiztostsi kifizetsek, a szocilis sszkiadsokon belli megntt az nkormnyzati seglyek rszarnya, de cskkentek a kzponti csaldtmogatsok s a munkanlkli seglyek. A munkanlkli jradk, az anyasgi tmogatsok s a szocilis seglyek egyre nagyobb arnyban jutnak el a legalacsonyabb jvedelmekhez. A nyugdjaknl ilyen nem volt megfigyelhet ezek a kifizetsek inkbb a kzps jvedelmi helyzeteknl koncentrldtak.

5.fejezet.Trsadalmi szerkezet s rtegzds


trsadalmi szerkezet: a trsadalmon belli klnbz pozcik kztti viszonyokat rtjk a trsadalmi szerkezeten. Konkrtabban a klnbz pozcikat betlt egynek s csoportok kztti viszonyokat jelenti. sttusz: a trsadalmi szerkezeten bell sttuszok vannak, a sttuszok betlti meghatrozott szerepek szerint viselkednek s ezek a szerepek meghatrozott viszonyokat rnak el. (Pl. A brmunks kteles a vezet utastsait kvetni.) trsadalmi rtegzds: a klnbz ismrvek mint foglalkozs,beoszts,munkahely,iskolai vgzettsg,lakhely alapjn megllaptott trsadalmi kategrik helyzetnek eltrse,hierarchikus sorrendje az letkrlmnyek s az letmd klnbz dimenziiban. A kategrik lehetnek az osztlyok,a rtegek vagy ms kisebb trsadalmi csoportok. Minden esetben hierarchikusan rendezdnek el. trsadalmi osztly: a termeleszkzkhz val viszony alapjn definilt trsadalmi kategria. Eredeti marxi rtelmezs szerint 3 osztly: 1. termeleszkzket nem birtokl munksok, 2. kisiparosok, kereskedk, nll parasztok 3. tksek, nagybirtokosok, akik brmunksokat foglalkoztatnak trsadalmi rteg: foglalkozs, iskolai vgzettsg, lakhely, a jvedelem nagysga stb. alapjn definilt trsadalmi

kategria. sttuszcsoport: Kolosi Tams (1984) defincija alapjn a sttuszcsoport az olyan egynek vagy csaldok csoportja, akik az ltala hasznlt ht dimenziban egytt vve hasonl helyzetben vannak. elit: a trsadalmi hierarchia cscsn elhelyezked kis ltszm az uralkod osztlynl szkebb csoportot.(A szociolgiban hatalmi elitrl beszlnk) presztzsvizsglat:(amerikai szociolgiban) trsadalmi szerkezet s rtegzdsvizsglatnak egyik mdszere. A megkrdezettek meg kell neveznik az egyes foglalkozsok presztzst. 3 vagy 5 fok skln rtkeltetik a klnbz foglalkozsokat s ennek tlagbl lesz a presztzs mrszma. (Magyarorszgon Kulcsr Rzsa(1985) vgzett orszgos presztzsfelvtelt.) trsadalmi gazdasgi sttusz: P.Blau s O.D. Duncan (1967) minden kis foglalkozsi csoport szmra meghatroztak egy trsadalmigazdasgi sttusz (SES) pontszmot s ezeket hasznltk fel a mobilitselemzseikben. kulturlis tke: P. Bourdie szerint 3 fle tke: pnztke,kulturlis tke s szocilis tke. A kulturlis tkn a mveltsget rti. Ezek a tkk egymsba konvertlhatk, tvlthatk. Pl. akinek j kapcsolatai vannak (szocilis tke) az knnyen jut vagyonhoz (pnztke). szocilis tke: P. Bourdie a trsadalmi kapcsolatokat rti ez alatt. A trsadalmi hierarchia cscshoz val feljutshoz nemcsak pnztke kell, hanem kultrlis s szocilis tke is,mert ezek nlkl nem hagyjk rvnyeslni az uralkod osztlyba. j osztly: Az j osztly M. Gyilasz (1957) jugoszlv kommunista politikus knyve,melyben azt rja,hogy a keleteurpai szocialista trsadalmak ugyanolyan osztlytrsadalmak,mint a kapitalistk,csak a szocialista trsadalmakban az llami s prtbrokrcia a termeleszkzk tnyleges tulajdonosa s ezltal az uralkod osztly. A szocialista orszgok eltltk Gyilasz josztly elmlett, de mgis nagy befolysa volt ezen orszgok szociolgusaikra. Vitakrdsek: 1.Inkbb az egyttmkds vagy a konfliktus jellemzi a trsadalmakat? A harmniaelmletek szerint: az emberi trsadalmakat alapveten a tagok egyttmkdse jellemzi,a konfliktus csak kivteles s diszfunkcionlis jelensg. Konfliktuselmlet szerint: minden emberi trsadalmat alapveten jellemzi a konfliktus,de ez nem felttlenl kros,mert a konfliktusok viszik elre a trsadalmi fejldst,konfliktus nlkl a trsadalom vltozatlan llapotban maradna. Csereelmletek szerint: a trsadalom tagjai egymssal javakat cserlnek,de nem csak anyagi javakat,hanem eszmei,szellemi javakat is.(szeretet,segtsg,megbecsls stb.) Knyszerelmletek szerint: a javak tads sokkal inkbb knyszer alatt trtnik,nem egyenrang felek tjn. Lenski szerint a harmniaelmlet hvei a konzervatvok, a konfliktuselmlet hvei a radiklisok. maga, a hatalmat tartja az egyenltlensg forrsnak. Konfliktuselmlet megfogalmazi: Marx s Engels 2. Mely fogalmak alkalmasabbak a magyar trsadalom szerkezetnek elemzsre a szocialista korszakban s a rendszervlts utn: osztlyok, rtegek, sttuszcsoportok, trsadalmi millik, trsadalmi helyzetek az elit s a hatalomnak alvetettek? 1945 eltt a magyar trsadalom: Az orszg lakossgnak a fele a mezgazdasgbl l,iparosods mg csak kezdeti stdiumban van a kzposztly rsz viszonylag kicsi,a trsadalom max 1/5t tette ki a trsadalom legszegnyebb rtegei a mezgazdasgi munksok,mezgazdasgi cseldek ,kisbirtokos parasztok jvedelem egyenltlensg nagyon nagy,nagyobb mint a korabeli Nmetorszgban vagy az Egyeslt llamokban Matolcsy szerint 2 vilghbor kztt Erdei Ferenc szerint ketts trsadalom ltezett M.oon: 1. trtnelmi nemzeti trsadalom (rendis jelleg),ahova az egyhzak,az llamhivatalnokok,trtnelmi arisztokrcia,nagybirtokosok,ri kzposztly,nemzeti kispolgrsg 2. modern polgri trsadalom,ide tartoznak a kapitalista vllalkozk,polgri szrmazs arisztokrcia,kapitalista kispolgr,kisipar s kiskereskedelem,szabad rtelmisgiek,polgri kzposztly,munkssg A parasztsg s a mezgazdasgi munkssg a trsadalom alatt l Erdei szerint. 1945tl kezdve 20 ven t a kt osztly egy rteg modell uralkodott a magyar trsadalomtudomnyokban. (sztlini modellnek is nevezik). Eszerint a szocialista trsadalmak szerkezett s rtegzdst gy lehet lerni,hogy abban a termeleszkzkhz val viszony alapjn 2 osztly: 1. munksosztly (llami tulajdonon keresztl rszesl a termeleszkzk tulajdonban) 2. termelszvetkezeti parasztsg (szvetkezeti tulajdonon keresztl rszesl a termeleszkzk tulajdonban) Ezek mellett kisebb maradk kategrik vannak: kisiparosok,1950es vek elejn pedig a kulkok,1960 utn pedig ide tartozott az egyni gazdlkod parasztsg is. Ferge Zsuzsa fle modell (1968): munkajelleg csoportok: 1. vezet s rtelmisgi 2. kzpszint szellemi (technikus) 3. irodai 4. szakmunks

5. betantott munks 6. segdmunks 7. mezgazdasgi fizikai munks vagy egyszerbben paraszt 8. nyugdjas A Ferge Zsuzsa fle modell s ennek tovbbfejlesztsbl, melyek a foglalkozsok jellege alapjn klntette el a trsadalmi kategrikat,rtegmodellnek,a kategrikat pedig rtegnek neveztk. Ferge Zsuzsa rtegzdsvizsglatbl s a ksbbi adatfelvtelekbl kitnt: a magyar trsadalomban lnyeges jvedelemegyenltlensgek vannak a szellemi foglalkozsak kategrijn bell lnyegesek a klnbsgek a vezetk, az rtelmisgiek s az irodai foglalkozsak kztt a munkssgon bell lnyeges klnbsgek vannak a szakmunksok s a szakkpzetlen munksok kztt a trsadalmi rtegek hierarchikus sorrendje az 1 fre jut tlagos jvedelem tern hasonlt a Nyugaton megfigyelt jvedelmi hierarchihoz, br a rtegek kztti jvedelem klnbsgek Magyarorszgon kisebbek 1980as vekben 2 elmleti modell szletett a magyar trsadalom szerkezetrl: 1. Szelnyi Ivn (1990): Abbl indul ki,hogy a msodik gazdasg s a piac fokozd szerepe lnyegesen megvltoztatja a trsadalom szerkezett.(Erdei Ferenc fle ketts trsadalomra visszautal) bra magyarzat: A nagyobb hromszg, aminek a cscsa is magas az llami redisztribci elvei szerint mkd gazdasgon alapul,a msik,a kisebb hromszg a piaci gazdasgon,ezen bell a msodik gazdasgon. Az elsnl rendi jelleg hierarchia rvnyesl,cscsn a kderbrkrcia ll,a msodik inkbb osztlyjelleg hierarchia cscsn a magnvllalkozkkal. 2. Kolosi Tams Lmodellje: Szelnyi modelljt gondolta tovbb. Mindketten arra utaltak,hogy a magyar trsadalom fejldse abba az irnyba halad,hogy a piaci hromszg megn s az llami redisztribci cskken.

A rendszervlts utn: A piac s a tbbprti demokrcia intzmnyein alapul trsadalom lett! A rtegek kzti klnbsgek nttek A trsadalmi sttuszok rendszere talakult,egyes sttuszok eltntek (prtkderek), msok megjelentek (munkanlkli,nagyvllalkoz) s az egyes sttuszokhoz tartoz kivltsok s htrnyok is megvltoztak. Fontos momentum: privatizcia piac s a redisztribci arnya 2/31/3re vltozott szemben a szocializmusban, ahol ez pont fordtva volt. Viszony nvekedett a munkanlklisg.> a munkjukat elvesztk egy rsze tartsan elhagyta a munkaerpiacot, inaktv lett Visszaesett: nehzipar, mezgazdasg,ami a szocializmusban jelents volt Megntt a multinacionlis vllalatok szma Magyarorszgon (klfldi tke beramlsval) Nagyvllalati szerkezetet felvltotta a tbb, kisebb vllalatszerkezetntt a munkanlkliek szma, nyugdjasok is viszont ntt a foglalkoztatottsg a szolgltatsokban: elssorban pnzgyi s humn szolg. terletn A jvedelmi s letkrlmny klnbsgek nvekedse a rendszervltozs ta: 2003ban Magyarorszgon a nagyvllalkozk,kzpvllalkozk valamint a szabad foglalkozs rtelmisgek trsadalmi helyzete a legjobb. Vagyon s fogyaszts alapjn: gazdlkodk,felss kzpvezetk,beosztott rtelmisgiek Laks krlmny szerint: fels s kzpvezetk,egyni vllalkozk Kolosi Tams s Rbert Pter olyan osztlysmt hoztak ltre, ahol a foglalkozsi pozcit s a trsadalmi sttuszhierarchiban elfoglalt helyet is magban foglalja. Ezek alapjn 5 csoport: Elit: (3%) nagy s kzepes vllalkozk,sttusindex fels tizedbe tartoz felsvezetk s rtelmisgiek Fels kzposztly: (8%) az elbbi kategribl kimaradt felsvezetk,a sttusindex fels tizedbe tartoz kzpvezetk,egyni gazdlkodk s vllalkozk,valamint a sttusindex 69. tizedbe tartoz rtelmisgiek Kzposztly: (31%) az eddig be nem sorolt kzpvezetket, gazdlkodkat,vllalkozkat, rtelmisgieket,alsvezetket,egyb szellemi foglalkozsakat,a sttusindex fels 3 tizedbe tartoz szakmunksok Munksosztly: (38%) az eddig be nem sorolt szakmunksok,szakkpzetlen s mezgazdasgi munksok kzl a sttusindex 4. vagy annl magasabb tizedbe kerlnek Deprivltak: (20%) a sttusindex als 3 tizedbe tartoz szakkpzetlen s mezgazdasgi fizikai munksok Az osztlyzsbl kimaradtak,akiknek sosem volt foglalkozsuk.

3.Kik alkottk az uralkod osztlyt a szocialista korszakban s kik alkotjk a rendszervlts utn? A szocialista korszakban a politikai s gazdasgi vezetket,brokrcit tekintettk az uralkod osztlynak. Konrd Gyrgy s Szelnyi Ivn 1970es vekben azt fogalmaztk meg,hogy az rtelmisg ton van,hogy uralkod osztlly vljon. Majd Szelnyi utn azt rta,hogy ez nem sikerlt az rtelmisgnek. Valjban a mai magyar szociolgia egyik legrdekesebb vitakrdse is ez: hogy a rendszervlts utn kik vlnak az uralkod osztlly: az rtelmisgiek,a vllalkozk,tksek,az llami brokratk vagy a magn s llami vllalatok,bankok menedzserei? 4. Elpolgrosodik-e a munksosztly a fejlett orszgokban s Magyarorszgon? 2 tudomnyos vita: Az els vita az 1960as vekben folyt arrl, hogy vajon elpolgrosodike a munksosztly. Az 1960-as vekben a msodik vilghbort kvet idszak kivteles kedvez gazdasgi konjunktrja, gyors gazdasgi nvekedse kvetkeztben NyugatEurpban s szakAmerikban a munksoknak egyre nagyobb rsze rte el azt az letsznvonalat, amely korbban a kzposztlyokat, a polgrsgot jellemezte: sajt hzban lakott, szemlygpkocsija, mosgpe, telefonja, televzija volt. Marx jslata a munksosztly elnyomorodsrl teht lthatan megcfoldott. A szociolgusok ltal felvetett krds ebben a helyzetben az volt, hogy a jmdban l" munks munkakrlmnyei, letmdja, gondolkodsa s nem utolssorban politikai vlemnye, prtpreferencija kzeledette a kzprtegekhez. A krdsnek konkrt politikai tartalma volt: ha ugyanis a munksosztly valban elpolgrosodik, akkor a szocialista s szocildemokrata prtok programjnak a politikai sznkp kzepe fel clszer kzelednie, hogy az elpolgrosod munksok szavazatait megtarthassk, Ellenttben az elpolgrosod munksosztly ttelnek kpviselivel, (Zweig 1961), J. Goldthorpe s D. Lockwood (1968; 1969) azt prbltk bizonytani, hogy a magas br s kzel irodai krlmnyek kztt dolgoz munksok sem vlnak polgrokk". Az angliai Lutton vrosban lv zemekben foglalkoztatott munksok hrom csoportjbl vettek mintt: az igen magas s korszer szakkpzettsg szakmunksokbl, a hagyomnyos szakmunksokbl s a betantott munksokbl. E munksokkal s csaldjaikkal ksztettek interjkat munkakrlmnyeikrl, letmdjukrl, aspirciikrl, a szakszervezetekhez val viszonyukrl s politikai nzeteikrl. A kutatsi krds az volt, hogy vajon a legmagasabb br s leginkbb fehrgallros" munkt vgz magas szakkpzettsg munksok kzelebb llnake a kzprtegekhez, mint a hagyomnyos szakmunksok s betantott munksok, vagy e hrom csoportot tovbbra is sszektik a tipikus munksjellemzk. Arra a kvetkeztetsre jutottak, hogy a munksok ltalban, s azokon bell a legmagasabb szakkpzettsg munksok is, inkbb munksok maradtak, mint elpolgrosodtak, teht megcfoltnak lttk a munksosztly elpolgrosodsnak tzist Kvetkeztetseiket utbb sokan brltk, s mint az 1990es vekben megindult msodik vita is mutatja, a tnyleges fejlemnyek is megkrdjeleztk lltsaikat. S. M. Ljpset s szerztrsai az 1990es vek elejn az International Sociology folyiratban feljtottk s hatrozottabban fogalmaztk meg azt a mr korbban msok ltal is kpviselt vlemnyt, hogy az osztlyok elhalnak vagy enyhbben fogalmazva jelentsgk cskken a trsadalom tagjai viselkedsnek, elssorban politikai llsfoglalsainak meghatrozsban (ClarkLipset 1991; ClarkLipsctRenipel 1993). rvelsket arra a fejlett orszgokban szles krben megfigyelt tnyre alapoztk, hogy a klnbz prtok elssorban a jobbkzp prtok s a szocildemokrata prtok szavaztbornak trsadalmi sszettele egyre kevsb klnbzik. Egyre tbb munks szavaz jobbkzp prtokra, pldul Angliban a Konzervatv Prtra, s egyre tbb, a kzprtegekhez tartoz szemly, elssorban rtelmisgi s llami alkalmazott szavaz a szocildemokrata prtokra, gy pldul Angliban a Munksprtra. Amennyiben ez a tendencia valsgos (amit egyes szociolgusok vitatnak), Goldthorpenak s Lockwoodnak a 25 vvel ezeltt a jltben l munks" vizsglat alapjn megfogalmazott kvetkeztetse tveds volt. Lipset s szerztrsai a munksok egy rsze jltnek emelkedsn kvl tbb ms magyarzatot adnak el a munksosztly fent lert preferenciira: 1. a jlti llam juttatsai, azltal hogy minden llampolgr megkapja ket, enyhtettk az osztlyellentteket, 2. a csaldi szrmazs fokozatosan kevsb hatrozza meg a trsadalmifoglalkozsi letplyt (a szociolgusok egy rsze ezt is vitatja), 3. a mai technolgia szmos olyan szakmt, foglalkozst hozott ltre, amely nem hasonlt a hajdani munksfoglalkozsokhoz, ugyanakkor nem is tipikusan hagyomnyos kzposztlybeli foglalkozs, 4. a szolgltatsok terletn foglalkoztatottak szmnak nvekedse olyan rteget vagy rtegeket hozott ltre, amelyek sem a munksosztlyhoz, sem a kzposztlyhoz nem tartoznak, 5. a nagy gyrak mellett, ahol nagy tmeg munks dolgozott egytt, nagyobb szerephez jutottak a kis s kzpvllalatok, ahol a munksok kisebb szma miatt kisebb az esly a munkstudat s sszetartozsrzs kialakulsra, 6. az osztlytudat s az osztlyharc kzppontjban a materilis nekek lltak, ma azonban egyre fontosabbakk vlnak a posztmaterilis rtkek, amelyek nem a munkskzposztly hatrvonal mentn vlasztjk szt a trsadalmat, 7. egyre inkbb rtelmt veszti a politikai nzetek s prtok jobboldalbaloldal szerinti klnvlsa, amely a hagyomnyos kzposztlymunksosztly politikai differencildsnak alapja volt.

Lipsetnek s szerztrsainak ezek a tanulmnyai nagy vitt vltottak ki, amelyben sem az elhal az osztly", sem a megmaradnak az osztlyok" llspont nem kerekedett fell. Azt mindenesetre mindkt fl elismerte, hogy a hagyomnyos egydimenzis osztlyfogalom nem alkalmas mr a mai fejlett trsadalmak szerkezetnek elemzsre. Nem alakult ki tbbsgi egyetrts abban a krdsben, hogy a hagyomnyos osztlyok helyett milyen kategrikkal clszer a mai trsadalmakat vizsglni, gy jelenleg klnfle szerkezetmodelleket hasznlnak, s kzttk az EriksonGoldthorpefle sma alkalmazsa a leggyakoribb. 5. Mennyire befolysolja a trsadalom tagjainak leteslyeit s gondolkodst az osztlyhelyzetk a mai Magyarorszgon? Valjban csak akkor van rtelme a trsadalmi szerkezetben elfoglalt pozcit vizsglni, ha annak nemcsak elmleti jelentsge van, hanem a trsadalom tagjainak letben megfigyelhet hatsa is. Max Weber ezt gy fogalmazta meg, hogy a trsadalmi pozci meghatrozza az egyes emberek leteslyeit. Az leteslyek annak a valsznsgt jelentik, hogy az egyn lete folyamn magas vagy alacsony jvedelemhez jut, milyen iskolai vgzettsget szerez, illetve gyermekei milyen iskolai vgzettsget fognak szerezni, milyen valsznsggel kerl brtnbe (akr azrt, mert trsadalmi helyzettl fgg annak valsznsge, hogy bncselekmnyt kvet el, akr azrt, mert ettl fgg az is, hogy bncselekmny elkvetse esetn a rendrsg letartztatja, s a bntetbrsg eltli) stb. Tnylegesen az leteslyeket mg szkebben s konkrtabban is rtelmezhetjk, ugyanis a klnbz trsadalmi kategrik tagjainak vrhat lettartama nagyon eltr egymstl, pldul egy 30 ves segdmunks vrhat letveinek szma lnyegesen kevesebb, mint egy 30 ves rtelmisgi Lnyegesek az osztlyok, rtegek tagjainak fizikai s lelki egszsgi llapotban mutatkoz klnbsgek is. Ennl sokkal szlesebb krben is befolysolja a trsadalmi helyzet az egyni letet. Amerikai szerzk (Caplowitz 1963) kimutattak, hogy a szegnyek ugyanazokrt a javakrt gyakran magasabb rat fizetnek, mint a jmdak. Nem ignyel sok magyarzatot, hogy a trsadalmi helyzet nagymrtkben meghatrozza, hogy az egyn kikkel bartkozik, kivel hzasodik ssze. Amerikban azt is, hogy melyik egyhznak a tagja, sszefoglalan azt mondhatjuk, hogy a trsadalmi helyzet igen nagy mrtkben meghatrozza az egyn letmdjt, hogy mit olvas, milyen zent hallgat, milyen sportot z, hogy tlti vi szabadsgt stb. Vgl, de nem utolssorban a trsadalmi helyzet egyes korszakokban s trsadalmakban ersen befolysolhatja a politikai preferencikat. A problmt mr Marx vilgosan megfogalmazta a magnak val s a magrt val osztly fogalomprjval. A magnak val osztly", elssorban magnak val munksosztly objektv helyzete s ebbl kvetkez valsgos rdekei alapjn klnl el a tbbi osztlytl, de nincs ers osztlytudata s nem harcol kollektvn rdekei rvnyestsrt. A magrt val osztly", klnsen a magrt val munksosztly viszont tisztn ltja rdekeit s azoknak ellenttt a tksosztly rdekeivel, ers munksntudata van, ennek alapjn kollektvan harcol rdekeirt s az osztlytudat legmagasabb szintjn forradalomra is hajland. A magrt val munksosztly gy Marx szerint a kommunista prtot tmogatja, annak irnymutatst fogadja el. A XIX. szzad msodik felben s a XX. szzad els ktharmadban a demokratikus politikai rendszer orszgokban meglehetsen kvetkezetesen rvnyeslt az a tendencia, hogy a htrnyos helyzet osztlyok, elssorban a munksosztly tagjai a szocildemokrata vagy szocialista prtra, esetleg a kommunista prtra szavaztak, mg a felsbb rtegek s kzprtegek tagjai, valamint egyes orszgokban a parasztsg a konzervatv prtokra szavaztak. Ennek az sszefggsnek az erssgt jabban sokan ktsgbe vonjk. 6.Milyen kvetkezmnnyel jrhat az, ha a fels trsadalmi rtegek arnya lnyegesen kisebb, mint a legals trsadalmi rtegek? A trsadalmi szerkezet alakulst egyrszt a gazdasgi fejlds hatrozza meg, msrszt a politika is lnyegesen befolysolja. Erre plda: a kommunista prtok azt grtk, hogy a munksosztlyt teszik uralkod osztlly s ltrehozzk a trsadalmi egyenlsget! Tnylegesen egy szk rteg kezbe kerlt a hatalom. Ha a jvedelmi egyenltlensgek kisebbek is lettek, mint a piacgazdasgban,az egyb nagyrszt rejtett trsadalmi egyenltlensgek igen lnyegesek maradtak.>a trsadalmi szerkezet nagy talaktsitl rdemes tartzkodni. 7. Mikpp rintette az egyes iskolzottsgi s letkori csoportok eslyeit a gazdasgi rendszervlts? A rendszervlts legfontosabb folyamatai trtnelmi lptkkel mrve viszonylag rvid id alatt j sztak le Magyarorsz on tal an a kelet s kzpeurpai orszgokban. Mg akkor is, ha Magyarorszgon a rendszervlts idszakt tgan rtelmezzk s kezdett a nyolcvanas vek harmadik harmadra (nagyjbl 1987 88ra, amikor a politikai vltozsok felgyorsultak, a gazdasgban j trsasgi trvny rakta le a piacgazdasg alapjait s j adrendszert vezettek be), vgt pedig nagyjbl az unis csatlakozs idejre tesszk, mintegy tizent ves folyamatrl beszlhetnk. Ez az idszak az emberi aktv letplynak csak egy szakasza s attl fggen, hogy az egyes trsadalmi csoportok tagjai letplyjuk melyik szakaszban voltak, nagyon eltr mrtkben hathatott rjuk a gazdasgi rendszer megvltozsa. Kolosi Tams s Rbert Pter (2004) ebben az rtelemben ngy csoportot klnbztetett meg a 2003ban legalbb 16 vesek kztt. Elveszett nemzedknek" neveztk azokat, akik 1988 ban mr nem voltak aktvak a munkaerpiacon (mintegy 16 szzalkra becsltk az arnyukat) s kilp nemzedknek" azokat, akik az 1988 s 2003 kztti idszakban elhagytk az aktvak munkaerpiact (kb. 33 szzalk). Aktv nemzedknek" azokat tekintettk, akik (legalbbis letkoruk s az idszak kezdetn s vgn betlttt munkaerpiaci sttusuk szerint) vgig aktvak voltak (kb. 27 szzalk) s belep nemzedknek" azokat, akik a szban forg idszakban vltak aktvv (kb. 18 szzalk). Rajtuk kvl mintegy 6 szzalk tartozott azoknak a krbe, akik az adatfelvtel idszakban mg tanulk voltak, gy termszetesen nem tudtak aktivizldni". Eredmnyeik szerint trsadalmi sttuszuk szerint a legjobb helyzetben az aktv nemzedk tagjai voltak, klnsen azok, akik vgig jelen tudtak lenni a munkaerpiacon. Kzttk a munkaerpiac szleire kerls (hosszabb rvidebb munkanlklisgi peridusok) egyben a deprivci megnvekedett veszlyt is jelentettk. Az elveszett nemzedk" tagjai zmmel folyamatos lecsszst ltek meg, kzttk mintegy ktharmadnyian voltak azok, akik a trsadalmi sttuszhierarchia als rszben helyezkedtek el. A kilp nemzedk" tagjai kzl sokan kerltek a deprivltak csoportjba, fggetlenl attl, hogy demogrfiai okokbl (letkoruk elrehaladsa miatt) vagy

munkaerpiaci okokbl szorultak ki. A belpk" lethelyzete nagyon heterogn kpet mutat: sszessgben nem olyan j a trsadalmi sttuszindexk, mint a vgig aktvk, de ebben szmos olyan ok jtszhat szerepet, amelyek relatv slyt csak ksbb lehet majd meghatrozni, hiszen csak ksbb derl ki, hogy idleges lemaradstl vagy az letplya egszben fennll htrnyrl beszlhetnk. Korbban sz volt arrl, hogy a rendszervlts idszakban a munkaerpiacon jelents letkorikereseti trendezds zajlott le. Mindez nyomon kvethet volt a hztartsok jvedelmi helyzetnek alakulsban is (Tth 2005). A hztartsfk jellemzi alapjn hrom letkori csoportot s ngy iskolzottsgi kategrit klnbztethetnk meg. Ennek megfelelen vizsglhat, hogy az egyes letkori/iskolzottsgi csoportok (pldul fiatal alacsony iskolzottsgak, idsebb kzpfok vgzettsgek stb.) tlaghoz mrt jvedelmi helyzete mikpp vltozott 1987 s 2003 kztt. Megllapthat, hogy: 1. letkori korszakon bell jelentsen nttek az egyes iskolai vgzettsgi szintekkel elrhet jvedelmek klnbsgei; 2. az azonos iskolzottsgi szinteken bell a vltozsok eltren rintettk az egyes letkori korszakokat; 3. az alacsonyabb iskolzottsgak kztt a fiatalok helyzete romlott, a magasabb iskolzottsgak kztt pedig ppen, hogy a fiatalok helyzete javult a leginkbb; 4. sszessgben mindez azzal jrt, hogy az alacsonyabb vgzettsg hztartsfk csoportjban meredekebb lett az letkorjvedelem profil, a felsfok vgzettsgek kztt pedig cskkentek az letplyaklnbsgek.

6. fejezet. Vros s falu


vros: Olyan telepls, melynek lakossga s krnyezete a krnyez falvak szmra bizonyos kzponti funkcikat lt el - pl. szakzletek, specializlt orvosi ellts, magasabb szint iskolk s mveldsi intzmnyek -, tbbnyire a krnyk llamigazgatsi kzpontjai. Az emberi kapcsolatok szemlytelenebbek, gyengbb az egyni viselkeds trsadalmi kontrollja. urbanzci: Magyar megfelelje a vrosods s vrosiasods. Vrosods az a tendencia, hogy a vrosi npessg arnya n, a vrosiasods a teleplsek vrosias jellege emelkedik, pl., n az emeletes pletek, kemny burkolat utak arnya, javul a kereskedelem. szuburbanizci: A vrosok kzponti rszeiben lak npessg szma fogy, a kls kerletek npessge, kertescsaldihzas terletek npessge n. szegregci: Egy teleplsen bell a klnbz trsadalmi rtegek etnikai csoportok stb. lakhelye ersen elklnl egymstl. Egytt jr a jvedelmi viszonyok s teleplsi infrastruktra lnyeges egyenltlensgeivel. slum: Vrosok, nagyvrosok fizikailag leromlott llapot s szegnyek ltal lakott vrosrszeinek megnevezse (ms szval: nyomornegyed). invzi: Egy teleplsbe, vrosi kerletbe a korbbi npessgtl eltr npessg kltzik be, a korbbi tevkenysgek mell jabb tevkenysgfajtk teleplnek. szukcesszi: Az invzi s a korbbi lakossg kikltzse kvetkeztben a teleplsrsz lakossgnak s gazdasgi tevkenysgeinek sszettele megvltozik, kicserldik. filtrci: A szukcesszival rokon fogalom, a laksszociolgiban hasznljk, azt jelenti, hogy az idk folyamn a laksok laki kicserldnek, azaz a laksokba egyre szegnyebb rtegek kltznek be, mg a korbbi lakk fokozatosan nagyobb s jobb minsg laksokba kltznek. Kondratyev-ciklus: Erre nem talltam magyarzatot ebben a fejezetben. A Wikipdibl tltttem le a kvetkezket, hogy tudjtok mirl van sz, de szerinemt a szrkvel jellt rsz elegend. A Kondratyev-ciklus vagy K-hullmok egy kzgazdasgi elmlet, ami a szoksos 1015 ves gazdasgi ciklusokon kvl aminek hatsait tbb-kevsb sikeresen cskkenteni prblta az 1990-es vekben megsznt (pl. Magyarorszg 1989) llami tervgazdasg, illetve a Franklin D. Roosevelt-fle New Deal-bl szrmaztathat modern polgri, magntulajdonon alapul trsadalom egy hosszabb, 4080 ves ciklus ltezst lltja. (Az eltr forrsok eltr ciklushosszakat emltenek). A Kondratyev-ciklus lte nehezen bizonythat vagy cfolhat, mivel a ciklusok hosszak, s a kapitalista (rtsd: magntulajdonon alapul polgri trsadalom, ma Magyarorszg is ilyen) vilgrend viszonylag fiatal, mindssze 4-6 teljes ciklus trtnhetett eddig. A Kondratyev-ciklus hajtereje az j tallmnyok (tallmny csaldok) bevezetse, elterjedse s kifutsa. 1. ciklus: A gzgp :1770-as vek 1830-es vek 2. ciklus: A vastptsek kora: 1830-as vek 1870-es vek 3. ciklus: Az elektromossg s a nehzipar kora :1870-es vek 1900-as vek 4. ciklus: Az olaj s az autgyrts kora: 1900-as vek 1970 es vek 5. ciklus: Az informatika, lzertechnika kora :1970-es vek 2008 ? 6. ciklus: Az rkorszak? kora : 2008 ? - ? A Kondratyev-ciklusra jl illeszkedik a 2008-as gazdasgi vilgvlsg. Tblzatba foglalva az eltelt vek alapjn kt nagy ciklus is kibontakozni ltszik: 17701900, valamint 19002030? (ez 2130 v lenne). A trsadalmi-gazdasgi folyamatok emberi mrtkkel mrve igen hossz ciklusidkkel brhatnak. szocilis vrosrehabilitci: A helyi lakossg helyben maradst lehetv tev, az ptett krnyezet feljtsn tl munkalehetsget biztost, a kzssgi funkcikat fejleszt vrosfeljts globlis vros: A vilgvrosok jelentsgt nem npessgk, hanem egyb jellemzik alapjn mri. A vrosok jelentsgt a vilggazdasgban betlttt szerepk alapjn hatrozza meg (pl. New York, London, Tokio).

Vitakrdsek 1. Merre tart az urbanizci a vilgon? A fejlett orszgokban az utols vtizedekben a vrosi npessg nvekedse lelassult, a fejld orszgokban viszont tovbb nnek a vrosok, kzttk sok a 10 millisnl nagyobb vros. Itt a npsrsg igen magas, a falusi szegnysg ell sokan a vrosokba meneklst vlasztjk, ahol alkalmi munkbl remlnek meglni. A fejlett orszgokban 1960 utn szuburbanizcis tendencia mutatkozott: a nagyvrosok lakossgnak egy rsze, elssorban a legjobb md csaldok elkltztek a vrosok kzponti rszeibl, s a vros krli kertes-csaldihzas vezetben telepedtek le. Ezzel egytt jrt a bels vrosrszek infrastrukturlis leromlsa, lakossguk elszegnyedse Elssorban a nagyvrosokban a lakossg sszettele s a jellemz tevkenysg alapjn (zleti negyed, kereskedelem, ipar, kultra) vezeteket lehet megklnbztetni. A slumosods ksrjelensge a szegregci ersdse. Az utols 10-15 vben j tendencia jelentkezett: nemcsak a nagyvrosok krli falusias terletek npessge nvekedett gyors temben, hanem a tvolabbiakban is megllt a lakossg cskkense, st kisebb nvekeds is mutatkozott, mert a korbbi nagyvrosi lakosok kezdtek ide is kikltzni. Az 1980-as vekben viszont jra az urbanizci jeleit vlik felfedezni -Amerikban, sok nyugat-eurpai orszgban s Japnban is, br a vrosok nvekedsnek teme nem annyira gyors, mint az 1950-es vekben. 2. Elmaradt-e Magyarorszgon az urbanizci az iparosods mgtt? Konrd Gyrgy s Szelnyi Ivn (1971) azt a ttelt fogalmaztk meg, hogy a szocialista orszgokban a vrosfejlds ksleltetve ment vge, mivel a kormnyzatok a termel elssorban ipari beruhzsoknak adtak prioritst, ezekkel szemben mindig httrbe szorultak az infrastruktrt, ezen bell a vrosi infrastruktrt fejleszt beruhzsok, gy tbbek kztt a lakspts is. Ennek kvetkeztben a vrosi npessg lassabban ntt, a vrosban az ipari zemekben foglalkoztatottak jelents rsze nem a vrosokban, hanem a krnyez kzsgekben lakott, s onnan ingzott naponta vagy annl ritkbban a vrosi munkahelyre. Mg inkbb megnyilvnult ez a politikai szndk a vrosokat igazn vross tv beruhzsok elhanyagolsban, ezrt a meglv vrosok is sok tekintetben falusiasak maradtak, nem voltak kpesek a vrosi funkcikat elltni. A falvakban viszont alig kerlt sor infrastrukturlis beruhzsokra, a lakspts majdnem teljesen magnerbl valsult meg, gy a kzsgek falusias jellege alig vltozott. 3. Melyek a leghtrnyosabb helyzet teleplsek Magyarorszgon? Az elmaradt rgik elssorban a keleti-szakkeleti rgi. Mikrotrsg tekintetben: Szabolcs-Szatmr Bereg megye keleti rsze s Borsod-Abaj Zempn megye szakkeleti rsze, Ngrd s Heves megye szaki rsze, JszNagykun-Szolnok megye szakkeleti rsze, tovbb Baranya, Somogy s Zala megye hatr menti kisfalvas rszei. Kiemelkeden kedvez Budapest, a kzponti rgi, az szaknyugat-dunndli s dli rgi. 4. Mi lehet a falvak jvje Magyarorszgon? Mi lesz a falvak jvje s hogyan lehet a kzsgek fejldst gy befolysolni, hogy rtkeik ne menjenek veszendbe az elnpteleneds kvetkeztben? A teleplsek fejldst kormnyzati s helyi llamigazgatsi dntsekkel lehet befolysolni. Gazdasgi cl, hogy a teleplsfejleszts alkalmazkodjk a gazdasgi fejlds kvetelmnyeihez, teht azokat a teleplseket fejlesszk, amelyek gazdasgi, fejldsi adottsgai a legjobbak. A szociolgusok a trsadalmi egyenltlensgek mrsklst s a szegregci cskkentst tzik ki clul. A politikusok hajlamosak ara, hogy adminisztratv eszkzket alkalmazzanak, pl. megtiltsk v. megneheztsk bizonyos teleplsekre val bekltzst, v. erszakkal kltztessenek be embereket bizonyos teleplsekre. Ezek az intzkedsek tbb krt okoznak, mint hasznot. Sokkal hatkonyabb gazdasgpolitikai eszkzk segtsgvel preferencikat biztostani a fejleszteni kvnt terleteken (j munkahelyek ltestse, meglv fejlesztse). Ennl is lnyegesebb a teleplspolitikai eszkznek mondhat fejlesztsi forrsok elosztsa. (infrastruktra fejlesztse). Sajnos ma mg inkbb csak a fvros s nhny kiemelt terlet kap arnytalanul sokat, az elmaradott terletek pedig arnytalanul kevesebbet kapnak. Helyi adk s ms befizetsek formjban (amelyek a teleplsen maradnak) lehetne nvelni a fejlesztst. 5. Mi lenne ma Magyarorszgon a legjobb lakspolitika: a magntulajdonban lv csaldi hzak s trsashzak ptse, vagy a magntulajdonban lv nagy brhzak ptse, vagy a jelentkeny llami tulajdon laksllomny fenntartsa s j llami lakspts a piaci rnl olcsbb lakbrek mellett? A lakspolitikban hrom alternatva klnbztethet meg: - A magntulajdonban lv csaldi hzak s trsashzak ptsnek elnys hitelekkel trtn sztnzse, - a magntulajdonban lv nagy brhzak ptsnek elsegtse, ehhez azonban teljesen szabadd kell tenni a lakbrek megllaptst, tovbb biztostani kell a tulajdonosoknak a kilakoltats jogt a nem fizet brlkkel szemben, - legalbb minimlis llami s nkormnyzati laksllomny s j lakspts fenntartsa, azaz alacsony lakbr laksok juttatsa az arra rszorulknak. Ebben a pillanatban nem ltszik mg, hogy a kvetkez vek magyarorszgi kormnyai melyik politika mellett fognak dnteni, illetve a hrom alternatv politika milyen kombincijt fogjk alkalmazni. A jelenlegi lakspolitika (pontosabban lakspolitika-hiny) fenntartsa mindenkppen slyosbtan a lakshinyt, nagymrtkben nehezten az j csaldok lakshoz jutst s lesten a trsadalmi egyenltlensgeket, mert a szegnyek lakshelyzete tovbb romlana. 6. Milyen rtelemben beszlhetnk a vros s falu kztti klnbsgek cskkensrl a kilencvenes vekben?

A 90-es vek egyik fontos s ritkn hangslyozott eredmnye, hogy a hagyomnyos infrastruktra igen gyorsan fejldtt a vidki teleplsek jelents rszn. Mindenkppen kiemelend a hlzati vzvezetk, a csatorna s gzhlzat fejlesztse.

8.fejezet. Npessg, npeseds, egszsggyi ellts


Demogrfia: a npessgszmmal s npesedsi folyamatokkal foglalkoz tudomny. 1662-tl szmtjuk a szletst, ekkor jelent meg John Graunt knyve, melyben a londoni hallozsi statisztikai adatok alapjn a halandsg trvnyszersgeit s ms npesedsi folyamatokat vizsglt. A legtbbet emlegetett demogrfus: John Malthus. Termkenysg: a szletsek vizsglatnl hasznljuk ezt a fogalmat. A szlkpes korban lv ni npessgre vagy annak egyes rszeire jut szletsek szmt rtjk alatta. Halandsg: ez alatt nem a hallozsok szmt rtjk, hanem valamilyen finomabb mutatjt, pldul a korcsoportonknti valsznsgt vagy mg inkbb a halandsgi tbla mutatit, elssorban a szletskori vrhat lettartamot rtjk. Reprodukci: a npessg jratermeldst jelenti, vagyis azt, hogy a halandsg s a termkenysg egyttes hatsa alatt a npessg szma s sszettele hossz tvon hogyan alakul. Morbidits: a betegsgek elfordulsnak gyakorisga. 2 fle mutatja van: incidenciamutatk, amelyek azt fejezik ki, hogy hnyan betegednek meg, illetve a prevalenciamutatk, amelyek pedig azt fejezik ki, hogy hnyan vannak beteg llapotban. Egszsg: a WHO alapokmnya szerint: a testi, szellemi s trsadalmi jlt teljes llapota, s nemcsak a betegsg s rokkantsg hinya. Egszsgesnek azt az embert tekintjk, akinek az letmkdsei az adott krnyezetben kiegyenslyozottak, lelki lete harmonikus, s a trsadalom vele szemben tmasztott elvrsainak kpes megfelelni. Mdszerek: Npszmllsok: Magyarorszgon 1784-87-ben volt az els npszmlls. 10 vente trtnik npszmlls. Magyarorszgon 1876 ta kzlik az vi szletsek, hallozsok, hzassgktsek adatait. Npmozgalmi statisztikk Reprezentatv mints, krdves mdszer Ezek alapjn sokfle s rszben igen finom szletsi mutatt szmtanak ki. A szletsek mutati: Nyers lveszletsi arnyszm: az vi szletsek szma osztva az vkzi npessggel. ltalnos termkenysgi arnyszm: az vi szletsek szma osztva a 15-49 ves nk szmval. Korspecifikus termkenysgi arnyszmok: az adott korcsoportba tartoz nk ltal szlt gyermekek szma osztva az adott korcsoportba tartoz nk szmval. Teljes termkenysgi arnyszm: a korspecifikus termkenysgi arnyszmok sszege a 15. ves kortl az 50. szletsnapig, teht a hipotetikus gyermekszm, amelyet egy n vagy ezer n szlne lete folyamn. Brutt/Nyers reprodukcis egytthat: a teljes termkenysgi arnyszm, de csak a lenyszletsek figyelembevtelvel, egy nre vonatkozan kiszmtva. Nett/Tiszta reprodukcis egytthat: a brutt reprodukcis egytthat a 0-14 ves csecsem-s gyermekhalandsggal s a 15-49 ves kor ni halandsggal cskkentve. Hallozs mutati, halandsgi tbla F forrsa az vi npmozgalmi statisztika. Nyers hallozsi arnyszm: az adott vi hallozsok szma osztva az vkzpi npessg szmval. Ez az arnyszm valban nagyon nyers, mert a hallozsok szma ersen fgg a npessg korsszetteltl, ugyanis egy olyan trsadalomban, ahol tbb az ids emberek szma, magasabb a nyers hallozsi arnyszm. Korspecifikus hallozsi arnyszmok: ezek az egy-egy tves vagy ves korcsoporton bell meghaltak szmt viszonytjk a krdses korcsoport ltszmhoz. Mivel a frfiak hallozsa kedveztlenebb a nknl, ezrt a kt nemre vonatkozan kln szoks ezeket az arnyszmokat kiszmtani. Halandsgi tbla: a halandsgi viszonyok tmr jellemzsre a szletskor vrhat tlagos ltettartamot szoks kiszmtani. gy szmtjuk ki, hogy az adott vi korves hallozsi adatokbl indulunk ki, ezeket azonban nem az vkzi npessghez viszonytjuk, hanem azt hatrozzuk meg, hogy az adott letkort meglk milyen valsznsggel halnak meg a kvetkez szletsnapjuk eltt. Ez a qx hallozsi valsznsg. Ezek alapjn meghatrozzuk, hogy egy hipotetikus 100 ezer szemlybl ll nemzedk, hogyan hal ki, ahogy letkora emelkedik, vagy megfordtva, a 100 ezer emberbl hnyan lesznek mg letben 1,2,3 .100 stb. ves korukban. Ez az lx tovbblsi rend. Vgl minden letkorban az azt kvet tovbblsi rend alapjn ki lehet szmtani, hogy az ilyen letkort meglt szemly tlagosan hny vig fog mg vrhatan lni. Ez az ex vrhat lettartam. Csecsemhalandsgi s termszetes szaporodsi arnyszmok Csecsemhalandsgi arnyszm: az sszes egy ven aluli korban meghaltak szma az adott vben osztva az sszes jszlttek szmval az adott vben. Demogrfiai tmenet elmlete: a halandsg hossz tv javulst s az azt kvet termkenysgcskkenst nevezzk demogrfiai tmenetnek. t egymst kvet fzisa: I. a hallozsi s a szletsi arnyszm egyarnt nagyon magas II. a halandsg javulni kezd, a termkenysg egyelre vltozatlanul magas marad a npessg gyorsan nni kezd III. a hallozsi arnyszm tovbb cskken, de a szletsi arnyszm is cskkenni kezd, a

npessg tovbbra is gyors IV. a hallozsin arnyszm cskkense lelassul, majd megll, a szletsi arnyszm tovbb cskken, ezrt a npessg nvekedse lelassul V. a hallozsi s szletsi arnyszm hasonlan alacsony szinten stabilizldik, a npessg nvekedse megll 1. Mikorra vrhat, hogy a Fld npessgnek szma stabilizldik, s az ekkorra elrt npessgszm okoz-e gazdasgi s politikai problmkat? A npesedsi tendencik kzl ma vilgszerte a vilg npessgnek igen gyors nvekedse kelt figyelmet. A vilg npessgszma a kzeljvben elri azt a hatrt, amelyet a Fld mg eltartani, lelmiszerrel elltni kpes. A nagy s nvekv npessgszm a termszeti krnyezet lerontshoz, st teljes pusztulshoz fog vezetni, tovbb hogy a gyors npessgnvekeds igen slyos nemzetkzi konfliktusokat eredmnyez a kzeljvben. A XVIII. szzadban indult meg a gyorsabb nvekeds elssorban Eurpban. Fld npessgnek alakulsa: Kr.e. 200 150 milli Kr.u. 1000 265 milli 1500-ban 425 milli 1700-ban 610 milli 1800-ban 900 milli 1900-ban 1625 milli 1950-ben 2500 milli 1994-ben 5,6 millird 2006-ban 6 millird Teht, ha a npessgszm a jvben is ilyen temben nne, akkor a nem is nagyon tvoli jvben elrkeznnk oda, hogy a Fld minden lakosra egy ngyzetmter szrazfldi terlet jutna. Teht valamilyen mdon a npessgnvekeds szksgkppen le fog lassulni, de nem mindegy persze, hogy a npessgszm hny millirdnl stabilizldik. 2. Milyen okai lehetnek a felntt frfiak halandsgromlsnak Magyarorszgon az 1960-as vek kzepe ta? 1960-ig gyorsan javult a halandsg, azonban az 1966. vi cscsrtk elrse utn a frfiak szletskor vrhat tlagos lettartama lnyegesen visszaesett (1993-ban 3 vvel volt kevesebb, mint 1966-ban). A frfiak halandsga a 25 ves letkortl flfel minden korcsoportban romlott, a legnagyobb fok romls a 40-54 vesek krben mutatkozott.A romlsnak krlbell a fele magyarzhat a szvkoszorr betegsgeinek (infarktus stb.) s az agyrbetegsgeknek (agyvrzs) a roml halandsgval, a fennmarad rszt a mjzsugorods, tdrk, az ngyilkossg s baleset okozta hallozs nvekedse idzte el. A legdrmaibb nvekedst a mjzsugorods (tszrs) s a tdrk (ktszeres) nvekedse okozta. Mindezek a megnvekedett alkoholfogyaszts, dohnyzs kvetkezmnyei. Az agyvrzsnek s a szivkoszorr-betegsgek okozta hallozsnak htterben pedig a magas vrnyoms ll. 3. Mi lehet az oka a halandsg nagy trsadalmi klnbsgeinek Magyarorszgon? A halandsg alakulsnak megrtshez tudnunk kell azt is, hogy trsadalmi rtegenknt igen nagyok a halandsg klnbsgei. A rossz egszsgi llapot s korbbi hall elssorban a fizikai foglalkozsakat, azok kztt is a htrnyosabb helyzeteket jellemzi. Nagy halandsgi klnbsgek vannak a terleti egysgek kztt is, elssorban a lakossguk trsadalmi sszettelvel sszefggsben. Pl. Budapest kerletei kztt akkork a halandsgi klnbsgek, hogy a jmd lakossg II. kerleti frfiak vrhat lettartama nagyjbl olyan, mint az NSZK frfinpessg, a szegny lakossg VII. kerleti frfiak viszont a szriai frfiakhoz hasonl halandsggal nznek szembe. Lehetsges tnyezk: A nem megfelel tpllkozs. A tpllk sszettele a trsadalom tbbsgnl tvolrl sem optimlis, mert a sznhidrt s zsr fogyasztsa az optimlisnl nagyobb, a zldsg s gymlcs fogyasztsa viszont az optimlisnl sokkal kisebb. Ennek oka minden bizonnyal az alacsonyabb jvedelmi szint. Munkahelyi egszsgi rtalmak. Lakkrnyezeti rtalmak. Hossz munkval tlttt id, hajszolt munkavgzs. Kevs szabadid. A magyar trsadalom tagjainak tbbsge egyltaln nem megy dlni. Az sszes emltett tnyez, valamint az emberi kapcsolatok feszltsgterhes volta kvetkeztben a magyar lakossg jelents rsznek lelki egszsgi llapota is problematikus. 4. A csaldok gyermekszma inkbb gazdasgi tnyezktl fgg , vagy inkbb a csald rtkeitl, normitl, gondolkodsmdjtl? Termkenysgelmletek: Kzgazdasgtani elmlet: chicagi iskola (G. Becker 1960) fogalmazta meg elsknt. Abbl indul ki, hogy egy hzaspr a gyermekek szmra vonatkoz dntsnl hasonlkppen gondolkozik, mint egy tarts fogyasztsi cikk esetben. Figyelembe veszi a gyermekekbl szrmaz hasznokat s rmket, a gyermeknevels kltsgeit, valamint a csaldi jvedelmet. A gazdasgi fejldssel emelkedik a csald jvedelme ezrt tbb gyermeket engedhet meg magnak, de ugyanakkor nnek a gyermeknevels kltsgei s esetleg cskkenek a gyermekekbl vrhat hasznok (ksbb vlnak kereskk). Az elmlet klnsen fontos szerepet tulajdont a nk keresetkiessnek, amelyet az anya lektttsge okoz. Mivel a nk kereseti lehetsgei javulnak (magasabb iskolzottsg rvn), ez a keresetkiess egyre jelentsebb. Vgeredmnyl, ebbl az elmletbl nem lehet levezetni, hogy a jvedelememelkeds milyen hatssal

van a gyermekszmra, de az egyrtelmen kvetkezik belle, hogy kt, egybknt azonos helyzet csald kzl a magasabb jvedelmnek lesz tbb gyermeke. Szociolgiai elmletek: a csaldok nem a gazdasgi helyzetk alapjn dntenek gyermekeik szmrl, hanem kvetik azokat az rtkeket s viselkedsi formkat, amelyeket trsadalmi krnyezetkben elfogadnak. A szociolgiai elmletek egyik alvltozata, a nk egyenjogsgra trekvsvel, karrierambcijuk nvekedsvel magyarzza. A gyermekvllals htrltatja az anyk elremenetelt a foglalkozsi karrierben. A hzassg felbomlsa esetn a gyermekek nemcsak az anya terheit nvelik, hanem az j partnerkapcsolatok ltestsnek a lehetsgt is cskkentik. Vgl a csonka (anya-gyermek) csaldok kztt is nagyobb a szegnyek arnya. 5. Indokolt-e, hogy a trsadalom politikai eszkzkkel prblja a csaldokat egy olyan gyermekszm megvalstsra sztnzni, amely a trsadalom egsznek hossz tv fejldse szempontjbl kvnatos? Igen indokolt, mivel jelenleg a npessgszm cskken tendencit mutat. A csaldi ptlk emelse s a gyermekgondozsi segly bevezetse a tbbi anyagi jelleg intzkedssel egytt legalbb meglltotta ezt a cskken tendencit. 6. Lehet-e a trsadalompolitikai eszkzkkel a csaldokat a gyermekszmra vonatkoz dntseikben befolysolni? Igen, mert a csaldok relatv anyagi helyzetnek javulsnak gyermekszmot nvel hatsa van. Msrszt sszeegyeztethet a termkenysg szociolgiai elmletvel kapcsolatban emltett azon megllaptssal, hogy a gyermekszmra vonatkoz trsadalmi normk nem egy bizonyos gyermekszmot kvnnak meg, hanem bizonyos jtkteret engednek, amelyen bell a csaldok gyermekszmt befolysolni lehet. A gazdasgi s trsadalmi felttelek vltozsa, tbbek kztt a csaldtmogatsok bevezetse, illetve emelse el kpes segteni a termkenysg 10-20 szzalkos emelkedst, viszont a felttelek romlsa tovbbi cskkenst indthat el. 7. Hatnak-e a csaldi ptlk s a hasonl anyagi juttatsok a csaldok tlagos gyermekszmra? Igen hatnak, gyermekszmot nvel hatsa lehet, ugyanis a csaldok a javul anyagi helyzetkre val tekintettel tbb gyermeket vllalhatnak. A csaldi ptlk s ms pnzbeli s kzvetett tmogatsok nvelse, amelyek a gyermekes csaldok helyzett knnytik, elsegtenk a termkenysg nvelst. Kt eurpai orszgban (Franciaorszg s Svdorszg), ahol a termkenysg cskkensi tendencija megllt, st bizonyos emelkeds is bekvetkezett, a gyermekes csaldokat tmogat jelents intzkedseket vezettek be. 8. Hat-e a mvi abortusz megengedettsge vagy tilalma, valamint a fogamzsgtl szerek hozzfrhetsge a szletsszmra? A mvi abortusz megengedettsge s a fogamzsgtl szerek hozzfrhetsge cskkentik a szletsszmot. 9. Melyek a msodik demogrfiai tmenet koncepcijnak alapvet jellemzi? Az j jelensgek, mint: alacsony termkenysg a hzassgon kvli szletsek megszaporodsa vlsok magas szma lettrsi kapcsolat elterjedse a hzasulsi kedv cskkense mgtt a mindent that individualizlds, a nemi szerepek klnbsgeinek olddsa, a nonkonformizmus terjedse s a fogyasztsi aspircik fokozdsa ll. A magas vlsi arnyszmok arra utalnak, hogy az egynek a partnerkapcsolatok minsgvel szemben magasabb ignyeket tmasztanak, s szles krben elterjedt az a nzet, hogy a kzs gyermek nem lehet akadlya a vlsnak. Erteljesen nnek a fogyasztsi ignyek, amelyeket korltoz a gyermekvllalsbl ered ktttsg s kltsg. Technolgiai s gazdasgi folyamatok segtik el az rtkvltozst. Annak nyomn, hogy hozzfrhetv vlnak a fogamzsgtls jabb, biztonsgosabb mdszerei, teret hdtanak a szexualits j formi, tovbb lazulhat a gyermekvllals s szexualits klcsnviszonya, megjulhat a nk iskolai kpzsnek idtartama, a ni foglalkoztats terjedse pedig kitolja a gyermekvllals kezdett. Ez tformlja a prvlaszts korbbi logikjt, kevsb szigorv vlnak az idbeli knyszerek, hosszabb egyeztetsi szakasz elzi meg a prkapcsolat rgztst. Vgl lehetv teszi nll ni letciklusfzisok kialakulst, egyes esetekben tartstja a szingli letformt, amely leggyakrabban gyermektelensggel is jr. A halandsg terletn azonban fokozatos javuls tapasztalhat. A cskken gyermekszm s a kedvezbb halandsgi viszonyok folyamnyaknt egyrszt Eurpa minden orszgban regszik a npessg, megn az ids korcsoportok arnya, negatvv vlik a termszetes szaporods arnya, teht a npessg fogy. Az ebbl ered htrnyok kezelsre jtt szba a migrci, konkrtan a helyettest migrci. 10. Miben klnbzik az 1990 eltti s azt kvet termkenysgcskkens Magyarorszgon? A rendszervlts a demogrfiai magatartsban is vltozst hozott. Ezek az 1990-es vek elejn kezddtek, de mg mig sem fejezdtek be. A termkenysget illeten a magyar npessg is elfordult a korai hzassg s korai gyermekvllals mintjtl, hiszen mg a nk 1990-ben tlagosan 23 vesen szltk els gyermekket, addig 2004ben 26,5 ves korukban. Noha elkpzelhet, hogy ilyen halasztst kveten is megszletnek a tovbbi gyermekek, de az ltalnos tapasztalat szerink az esly mindenkpp cskken. gy vrhat, hogy a jelensg terjedse nyomn kevesebb gyermek szletik Magyarorszgon, a csaldmodell pedig differencildni fog. A differencilds abban is megmutatkozik, hogy a gyermekek tbb mint egyharmada hzassgon kvl jn a vilgra, tbbsgk lettrsi kapcsolat keretei kztt, de nem elhanyagolhat a gyermekket egyedl vllal anyk arnya. A vltozsokat az albbi okok motivljk: az oktats expanzijnak hatsra elhalasztdik az els szls

fokozd anyagi ldozatok, a gazdasgi szkssg s a gyermekvllals megnvekedett kltsgei hatsra ltalnos bizonytalansg, ami kedveztlen krlmnyeket teremt a hossztv dntsekhez, pedig a gyermekvllals mindenkppen annak szmt fogyasztsorientlt rtkek terjedse, ami szemben ll a gyermekvllalssal hzassgktsek visszaszorulsa, gyermekvllalsi hajlandsg visszaesst okozza a csald s a munka sszeegyeztetse nem knny csaldpolitikai rendszer instabilitsa Az 1990-ig letbe lpett trsadalompolitikai intzkedsek hatkonysgt nem knny megtlni. Els ltsra gy tnhet, hogy nem jrtak igazi eredmnnyel, hisz csak ksleltetni tudtk a gyermekvllalsi hajlandsg visszaesst. Az rintettek a ksbbre tervezett szlsket az intzkedsek hatsra elrbb hoztk, viszont nem vllaltak tbb gyermeket.

11.fejezet.Csald
Trsadalmi intzmny: feladata: kzvett szerep az ember s a makrotrs-i struktrk kztt. Az egymssal sszefgg tevkenysgekre vonatkoz normk s rtkek rendszere, melyek a trsadalmi letet szervezik oly mdon, hogy a trsadalom tagjai szmra a szksges funkcikat ellssk. Ilyenek: csald, oktatsi rendszer, gazdasgi rendszer (vllalatok), kormnyzat, politikai intzmnyek. Norma: viselkedsi szably, mely elrja, hogy a trsadalom tagjainak bizonyos helyzetekben hogy kell s hogyan nem szabad viselkednik. Megszegst mindig valamilyen szankci bnteti. Sokfle norma rvnyeslhet egyms mellett: bntetjogi normk, erklcsi normk, szoksok, illemszablyok stb. Szerep: valamely sttushoz tartoz viselkedsi mintk, jogok s ktelezettsgek egyttese. Pldul az egyetemi oktat szerepnek normi: sznvonalas eladsokat tart, lelkiismeretesen vizsgztat, tudomnyos kutatsokat vgez, publikl. A trsadalom elvrja, hogy e sttusokat betltk megfeleljenek a velk szemben tmasztott viselkedsi elvrsoknak. Sttusz: az egyn ltal a trsadalomban s trsadalmi intzmnyekben elfoglalt pozci. Egy szemly tbbfle sttuszt foglalhat el. Pld. egyszerre frj, apa, gyermek, tanr, egyesleti vezetsgi tag. Minden sttushoz viselkedsi elvrsok kapcsoldnak. (lsd: szerep) Csald: egytt l kiscsoport, amelynek tagjait hzassgi vagy leszrmazsi, ms szval vrsgi (esetlen rkbefogadsi) kapcsolat kti ssze. Csald funkcii: (5) 1. termels, 2. fogyaszts, 3. demogrfiai reprodukci (gyerek), 4. A gyermekek szocializcija, 5. a felnttek pszichs vdelme. Csaldmag: ms nven nukleris csald. 3 tipus: 1. hzaspr, 2. hzaspr s gyermek, 3. Egy szl s gyermek. Hztarts: az egytt lak s a meglhetsi kltsgeket megoszt, egytt fogyaszt emberek csoportja. A hztarts tagjai nem szksgkppen egy csald tagjai vagy rokonok. Nukleris csaldi hztarts: olyan hztarts, amelyben csak egyetlen csaldmag tagjai lnek. (esetleg + hztartsi alkalmazottak) Kiterjesztett csaldi hztarts: olyan hztarts, amelyben a csaldmaghoz nem tartoz rokon vagy rokonok is lnek (+ hzt.alk) Tbb csaldmagbl ll hztarts: amelyben egynl tbb csaldmag l egytt (s esetleg egyb rokon s hzt.alk) Nyugat-eurpai hzassgktsi minta: Ny-Eu-ban a ks kzpkortl a 19. sz-ig rvnyeslt hzassgktsi minta, amely szerint viszonylag ksn hzasodtak s viszonylag sokan maradta letk vgig ntlenek ill. hajadonok. lettrsi kapcsolat: jogi cselekmny nlkli tarts csaldi egyttls Incestus: vrfertzs, Endogmia: falun belli hzassgkts Exogmia: falun kvli hzassgkts Homogmia: azonos trs-i oszt-on, rtegen belli, Heterogmia: -- kivli hzassgkts Trsadalmi csoportok nagy:osztly, rteg, - kis: akiknek tagjai ismerik egymst (csaldok, klubok, munkahelyi kzssgek) Formlis csoport: szervezetileg szablyozott, Informlis cs: szemlyek kapcsolatok, rdekek tartjk ssze. Elsdleges cs: rzelmileg szinezett, a tag teljes szemlyisgvel rszt vesz, szemlyisgk fejldst szolglja > a gyermek szocializcijnak sznhelye. Vitakrdsek 1. Melyek a csald funkcii s mennyiben vltozott a csald e funkcik betltsben? 5 funkci: 1. termels, 2 fogyaszts, 3. a npessg reprodukcija, 4. a gyermekek szocializcija, 5. a felnttek pszichs vdelme. Vltozs: a modern trsadalomban a hztarts s a munkahely sztvlt, ezrt termelsi funkci httrbe szorult, de nem teljesen. Mert br a termels csaldon kvl, vllalatoknl folyik, maradt mg jelents rtk csaldi termels: a mezgazdasgi termels kb. egyharmada csaldi gazdasgokban folyik, s a hzilagos kivitelezsek: lakspts, javts, szerels, hzimunka mint vendgli vagy mosodai szolgltats kivltsa a nemzeti jvedelmet nvel, csaldi keretekben vgzett termels. A fogyaszts annyiban vltozott, amennyiben a csald kztkeztetsben s tteremben tkezik elenysz mrtkben. A reprodukci a hzassgon kivli szlsek arnya ntt, de mr nem olyan jelents a vltozs, ha a hzassg-szer egyttleseket (tarts lettrsi kapcsolatok) nem szmtjuk ide csak a tiszta egyszls szletseket. A gyermek szocializcijban s a felnttek lelki tmogatsban pedig mg nagyobb szerepe van ma, mint rgen. A nevelsben szerepet jtsz ms trs-i

intzmnyek (nagy-rokonsg, szomszdi kzssgek, ifjsgi egyesletek) szerepe cskkent. A mindennapi let feszltsgeinek, kudarcainak elviselsre a csald nyjtotta vdelem semmivel nem ptolhat. Az idsek s lelki betegek gondozsra sem tudnak a trs-i intzmnyek alternatvt nyjtani. 2. Vltozott-e az rzelmek szerepe a hzassgktsnl az vszzadok folyamn? Trtnelmnk sorn a prvlaszts-szexualits tbb-kevsb rzelemmentes volt. Az 1700- as vekben indult meg az els szexulis forradalom, a romantikus szerelem , az rzelmek trhdtsa. 1960 az jabb szexulis forradalom, a hzassg eltti, akr tbb egymst kvet szexulis kapcsolatok kezdete. (Br egyesek felttelezik, hogy ez a vltozs ltez, de nem akkora mint sokan gondoljk, csak rgen sokkal jobban titkoltk.) 3. Vlsgban van-e a hzassg s a csald a fejlett orszgokban s Mo-n? Nemzetkzi s magyar adatok szerint egyarnt vlsgjelensgek: 1. az els hzassgktsi letkor kitoldik. 2. megntt a hzassgon kvl lk arnya. 3.ntt a hzassgon kvl szletettek arnya, (tizenvesek feleltlensg, ismeretek hinya, s skandinv tipus hzassgszer egyttlsben szletettek) 4.n a vlsi arnyszm, 5. ntt egyszls csaldok szma, 6. ntt az egyszemlyek hztartsokban lk szma. (tovbb lnk az egyedl l zvegyek szma n, s szinglijelensg) 7. cskken a csaldok tlagos gyerekszma. Szociolgusok kztt nincs egyetrts abban, hogy ez valdi vlsg-e. Elmletek: Easterlin: csak ciklikus jelensgek s a gazdasg rossz llapotval, a fiatalok nehezebb krlmnyeivel hozhat sszefggsbe. Becker: a ni emancipcira vezethet vissza, s visszafordthatatlan. Msok szerint: a modern trsadalom jellemzje, okai? rtkvltozsok, a kzssg tipus kapcsolatok httrbeszorulsa, a kapcsolatok elszemlytelenedse, anmia. Nmet kutatk: a posztmodern csald jellemzi: 1. az egyni letplya szabvnyossga megsznik (j tipus s ms sorrend letszakaszok: lettrsi kzssg, jrahzasods, gyerekek j csaldba kerlse, jrakisgyerekeseds idsebb korban,) s 2. a csaldi letformk pluralizldsa (j csaldtipusok: gyereknlkli csaldok, jrahzasodottak elz gyerekekkel s kzs j gyerekkel, ) Ez a folyamat felfoghat egyfajta talakulsnak, nem felttlen vlsg. Magyarorszgon a 90es vekben kt, a csalddal kapcsolatos rtkeket kutat vizsglat volt, mindkett eredmnye szerint a hagyomnyos rtkek s normk helyeslse volt tlslyban. A csaldtagok egymsrt hozott ldozata, a tekintly szerepe a gyereknevelsben, a kt szl + gyerek csaldforma helyeslse. A nemzetkzi felmrs szerint Mo. adta a leginkbb csald- s gyermekbartabb vlaszokat. 4. Mely tnyezk jrultak hozz a modern vilgban a vlsok terjedshez? 1. A prkapcsolatokat fenntart knyszerek megsznse (nk munkavllalsa, gazdasgi nllsga), 2. a prkapcsolat minsge (vonz elemek s feszltsgek arnya), 3. Morlis rtk-vltozs (a hzassg felbonthatatlansgba vetett hit) 4. alternatvk vonzereje (egy j prkapcsolat, nll let lehetsge). 5. Mi jellemzi a prkapcsolatok alakulst az 1990-es rendszervltozs utn Mo-n? Az talakuls mr 90 eltt megkezddtt, de azta felgyorsult. Els kapcsolatknt gyakoribb az lettrsi kapcsolat (36,6 %) mint a hzassg (32,3 %) Normavltozs: a fiatalok 75 %-a s a teljes lakossg 60 %-a preferlja az elszr letkzssg csak aztn hzassg elvt. A hzasodk letkora emiatt (is) n. Okok: kpzsek idejnek kitoldsa, az els munkahely megszerzsnek bizonytalansga. rtkvltozsok: vlsok szmnak nvekedse hatsra az rkk tart hzassgba vetett hit elvesztse, individualizlds, jelenorientlt viselkeds. Jellemzen a nem vallsosak, a fggetlensget kiemelt rtkknt kezelk, a szocializci sorn nehzsgekkel kzdk inkbb vlasztjk az lettrsi kzssget. DE: a kztudatban mg mindig hzassg mint vgs cl dominl a kztudatban. 6. Mi jellemzi a magyar trsadalom hztartsok szerinti sszettelt, s melyek a msodik VH utni leglnyegesebb tendencik? Hztartsnagysg alakulsa: II. Jzsef idejn: 5,28 f/hztarts., 1960: 3,1., 1970: 2,96., 1980: 2,79., 1990: 2,6. 1990-ben: 3 milli 890 ezer hztartsbl: 19,2 % ( 2 milli ember) tbb rokon csaldmag egyttlse (tbbgenercis egyttls), ez a hztartsok 12 %-a. Nyugat-Eurpval s szak- Amerikval sszehasonltva nlunk a legnagyobb ez a kiterjesztett s tbbcsaldmagos hztartsok arnya. A maradk eloszlsa: egy szemlyes hztarts (zvegy, szingli) ht 24,3 %- a, np 9,3 %-a, nukleris csald: ht 60 %-a, np 68,3 %-a. A tbbi: tbbszemlyes hztarts, melyben nincs egy csald sem, intzeti hztarts. 7. Milyen trsadalompolitikai eszkzkkel lehet a csaldokat segteni feladataik elltsban? Gyes, Gyed, Szlsi szabadsg, voda, blcsde, napkzi (jobb szinvonal), csaldi ptlk, csaldsegt kzpontok, lakshoz juts segtse, csaldi adrendszer, ids-gondozs, jobb egszsggyi ellts. 18.fejezet. Valls Valls: Durkheim szerint a szent dolgokra vonatkoz hitek s gyakorlatok egysges rendszere. Megklnbztet profn s szent dolgokat. Profn mindaz, ami az adott trsadalomban mindennapos, szoksos. A szent dolgok nem htkznapiak, meghaladjk a kzvetlen lttapasztalatot. Egyhz: Nagyobb ltszm trsadalmi szervezet, a tagsg nagyrszt beleszletik, elklnlt s kpzett papsga van. Az adott trsadalom teljes vagy majdnem teljes npessge hozztartozik, azltal az egyhz sszekapcsoldik az lammal. Szekta :Kisebb ltszm, a tagsghoz egyszeri felnttkori csatlakozs, megtrs szksges, tbbnyire nincs specializlt papsga, az rzelmi elemeket, a vallsos lmnyeket hangslyozza. Felekezet : Flton van a szekta s az egyhz kztt. Olyan egyhz, amely elfogadta, hogy tbbvalls trsadalomban l, tolerns a tbbi felekezettel szemben. Fggetlenek az llamtl, tbb felekezet l egytt a trsadalomban.

Kultusz : Formtlan vallsi vagy nem vallsi csoportosuls, amely valamely hit vagy ritul krl alakul ki (satanizmus, boszorknyhit). j vallsi mozgalom : Szekta s kultusz jelleg mozgalom, a valls fenti tg defincija szerint tekinthet vallsnak. Fundamentalizmus : Azok a vallsi mozgalmak, csoportok, kzssgek, amelyek ersen ragaszkodnak nhny leegyszerstett hitttelhez, utbbiakat hajlamosak rerszakolni a tgabb trsadalmi krnyezetkre, a ttelektl val eltrst hitetlensgnek, eretneksgnek tekintik. 1.Hat-e a valls a gazdasgi s a trsadalmi rendszerre? Vallsnak tekintnk minden olyan eszmerendszert, amely az 1 emberi s trsadalmi let vgs krdseire, az let rtelmre s cljra vonatkoz vlaszokat fogalmaz meg (akkor is, ha nem hivatkozik Istenre). sszessgben azt mondhatjuk, hogy minden emberi trsadalomban volt valls. Durkheim szerint a trsadalom s a valls elvlaszthatatlanok egymstl. A szociolginak elklnlt s intzmnyes ga a vallsszociolgia. A vallsszociolgia legalapvetbb krdse, hogy mi a valls szerepe a trsadalomban? Durkheim szerint minden trsadalomnak szksge van ritulkra, amelyek a trsadalom tagjai szmra tudatostjk azokat az rtkeket, amelyekre a trsadalom egyttmkdse s integrcija tmaszkodik. Berger s Luckmann szerint az sztnk nem kpesek a cselekedetek irnytsra. Az embernek viselkedsi szablyokat, normkat kell kialaktania a maga szmra. A normk, ha nem tmasztja al ket meggyzds, hogy valamilyen rtkekbl kvetkeznek, akkor gyenge lbakon llnak. Az rtkek tmaszai az emberi let vgs vagy egzisztencilis krdseire adott vlaszok. A vallsok utbbiakra knlnak vlaszokat. Ahhoz, hogy az ember az t krlvev vilgban tjkozdni tudjon, valamilyen vallsi jelleg elkpzelsre, vilgkpre van szksge. A vallsok segtenek az embernek, hogy tjkozdni tudjon a vilgban, az egsz trsadalom szmra valamilyen kzs rtelmezsi keretet nyjtanak az let rtelmrl, az rtkekrl s a normkrl. A valls elsegti a trsadalmi integrcit, az sszetartozs rzst, kzssgeket teremt. Marx szerint a valls a np piuma, a valls s az egyhz mindig a fennll viszonyok megmaradsban rdekelt erk oldaln ll, azokat szolglja. Max Weber munkssgnak jelents rszt alkotjk vallsszociolgiai mvei, amelyekben a nagy vallsok (pl.: hinduizmus, taoizmus) hatst vizsglta a gazdasgra s a trsadalomra. Ezekben azt mutatta ki, hogy ezek nagyon lnyegesen befolysoltk, hogy az adott trsadalomban milyen gazdasgi rendszer alakult ki. Nmely vallsok htrltattk, msok elsegtettk a modern (kapitalista) gazdasg fel irnyul fejldst. A valls a trsadalmi vltozst elremozdt hater lehet. 2. Lehet-e a folytatd szekularizcis tendencirl beszlni a vilgban? A vallsszociolgia msik kzponti elmleti krdse a szekularizci. Az els szekularizcis elmletek szerint a gazdasgi s trsadalmi fejldssel, a racionlis gondolkods elterjedsvel a valls fokozatosan elveszti fontossgt, szerepe gyengl, teljesen megszakad az emberek kapcsolata az egyhzzal, az egyhz befolysa s szerepe a politikban, az oktatsban, az egszsggyi elltsban fokozatosan megsznik. Durkheim szerint, ha van olyan igazsg, amelyet a trtnelem vitathatatlanul bizonyt, akkor az az, hogy a valls a trsadalmi letnek egyre kisebb rszt leli t. Max Weber a szekularizci helyett a vilg varzstalantsrl beszlt. Ezen azt rtette, hogy a modern trsadalmakban teret hdt a vallsi magyarzatokkal szemben a krnyezet, a termszet, a trsadalom racionlis magyarzata. jabban a szekularizcin nem a valls s az egyhz eltnst, inkbb vltozst rtik. A szekularizcin bell tbb egymssal csak lazn sszefgg folyamatot klnbztetnek meg.Ezek a kvetkezk: 1. A racionlis gondolkods elterjedse a vallsos gondolkodsban is ersdtt a racionlis elem 2. A nem sajtosan vallsi feladatok (oktats, betegpols) elvgzsnek elfggetlentsed az egyhzaktl. Elsegtheti, hogy az egyhzak a vallsi ignyekre sszpontostsanak. 3. talakul az egyhzak szervezete, gyengl a hierarchia, ersdik a kis spontn csoportok nllsga s a laikusok szerepe a vallsi tevkenysgben. Kevss valszn, hogy a szekularizci a valls s az egyhz elhalshoz fog vezetni. Az elmlt vtizedekben a fejlett orszgokban olyan j tendencik jelentkeztek, amelyek megkrdjelezik a szekularizci folytatdst. Ilyen a vallsos breds egyes orszgokban (Egyeslt llamokban), a fundamentalizmus megersdse, tovbb az j vallsi mozgalmak megjelense. 3. Mivel lehet a vallsossg vltozsait magyarzni Magyarorszgon? 1949. vi npszmlls volt az utols, amikor a felekezeti hovatartozst krdeztk. 1992 vgn a KSH mobilitsfelvtel keretben adatot gyjttt arrl, hogy a megkrdezetteket milyen felekezetben kereszteltk meg. 1930-tl 1949-ig a leglnyegesebb vltozs az izraelitk szmnak nagyfok cskkense a II. vilghbor alatti holokauszt s a kivndorls miatt. Az evanglikusok szmnak cskkense sszefgghet a nmet anyanyelvek kiteleptse, szlovkok egy rsznek tteleplse Szlovkiba. 1949-tl 1992-ig a leglnyegesebb vltozs a felekezeteken kvliek szmnak a nvekedse. A Tmegkommunikcis Kzpont 1972 ta rendszeresen feltette a krdst. Vallsos ember-e n? 1972-ben magas (46%) Folyamatos cskkens : 1978-ban (36%) 1990-ben (51%) A KSH 1992. vi felvtelbl a vallsgyakorls mdjra vonatkoz adatokat tallunk. A reformtusok krben ltszik a legelrehaladottabbnak a vallstl val elforduls, szekularizci. Visszaszorult a rgi, npegyhzi vallsossg, amelyben a fiatalok nagy rsze szinte termszetesen kerlt kapcsolatba az egyhzzal. 4. Hogyan jellemezhetjk a maguk mdjn vallsosak csoportjt?

Tompa Mikls a vallsossg mrtkt vizsglta 1980-tl. A megkrdezettek 5 alternatv vlaszlehetsg kzl vlaszthattak. Maguk mdjn vallsosak: 1980-ban 40,9% 1985-ben 44,8% 1990-ben 49,4% A magyar trsadalom felntt tagjainak egy viszonylag kicsi, de nvekv rsze vallsos abban a szigor rtelemben, hogy az egyhz tantsait kveti s rendszeresen jr templomba. 70-80% ezek kztt helyezkedik el, rszben kzmbs, valamilyen laza ktdst mutat az egyhzakhoz. 1980. s 1990. kztt lass eltoldds megy vgbe a vallsossg irnyban. 1990 utn nem tapasztalhat nvekedsi tendencia a magukat vallsosnak mondk kztt. Nagy a trsadalomnak az a rsze, amely a maga mdjn vallsos. 1990 utn Magyarorszgon is megjelentek j vallsi mozgalmak. 5. Milyen sszefggseket figyelhetnk meg az iskolai vgzettsg s a vallsossg kztt? A katolikus, reformtus, evanglikus s a felekezeteken kvliek iskolai vgzettsg szerinti sszettele azt jelzi, hogy leginkbb a magas iskolai vgzettsgek kztt tallunk felekezeten kvlieket. A hrom felekezet kzl az evanglikusok iskolai vgzettsge volt a legmagasabb. Korbban a vallsosan arnya az tlagosnl nagyobb volt a nk, az idsek s az alacsony iskolai vgzettsgek kztt. 1978 utn nvekedni kezdett a fiatalok s a magasabb iskolai vgzettsgek kztt. Terjed a passzv-befogad-fogyaszt tpus vallsossggal szemben az aktv-keresalkot tpus. Ez sszefgg a papok szmnak cskkensvel, a papok regedsvel.

17. fejezet. Devins viselkeds


anmia: = rtkvesztett llapot. Csaldban s trsadalomban az anmia kzs normk, kzs rtkek hinya, melynek kvetkeztben az ember nem tudja, mihez tartsa magt, mire szmthat. A szociolgia klasszikus devianciaelmletei az anmiaelmletek. Ezek magyarzzk ugyanis a makrotrsadalom jellemzivel, struktrjval, ellentmondsaival a devins viselkeds gyakorisgt. Egyben ezek azok az elmletek, amelyek a legkvetkezetesebben alkalmazzk azt a szemletet, hogy a devins viselkedsnek kzs gykereik vannak. devins viselkeds: Az emberek trsadalmi egyttlse azrt lehetsges, mert nagy tbbsgk alkalmazkodik az adott trsadalom elfogadott normkhoz, viselkedsi szablyokhoz. Ugyanakkor minden trsadalomban elfordul, hogy egyes tagjai ezeket a normkat megszegik. A normaszegst nevezik deviancinak, a normaszeg viselkedst devins viselkedsnek. elidegeneds: Az anmhoz hasonl fogalom az elidegeneds. E fogalmat Marx ta nagyon sokfle rtelemben hasznltk a szociolgiban. A marxizmustl tvol ll elidegenedsi fogalom Seeman (1959) kidolgozsa alapjn, melynek dimenzii: a hatalomnlklisg, az let rtelmetlensge, az elmagnyosods, az nmagtl val elidegeneds, az nrtkels elvesztse, vgl a munkanlklisg. alkoholizmus: Az alkohol rendszeres, mrtktelen, hozzszoksszer fogyasztsbl ered, testi s lelki krosodsokkal jr llapot, amely az alkoholfogyasztsrl val leszoks kptelensgvel trsul. Alkoholista az, akinl mr kialakult az n. dependendencia (fggsg), s ivsn nem tud uralkodni. Az egyik csoportba azok tartoznak, akiknek mindennap szeszesitalt kell fogyasztaniuk. A msik csoportba tartozk csupn nem kpesek megllni a teljes lerszegedsig, ltalban hetentekthetente htvgn, de nluk olyan is elfordul, hogy hnapokig nem isznak egy csepp alkoholt sem, aztn pedig napokig ittasak. epidemiolgiai felvtel: A kbtszer fogyasztk mrsnl problmk merltek fel. Bizonytalan a kbtszer fogyaszt defincija, mivel sokfle, klnfle veszlyessg kbtszer ltezik (pldul lteznek gygyszer dependencik), s nagyon eltr gyakorisggal s mennyisgben fordul el kbtszerfogyaszts. Becslsi mdszerek egyelre nem llnak rendelkezsre, tbbek kztt azrt sem, mert az sszes fogyasztsrl semmi adat sincsen. A lakossg megkrdezsn alapul epidemiolgiai felvteleket lehet vgezni, de a letagads eslye minden bizonnyal mg nagyobb, mint az alkoholfogyaszts esetben. kbtszer: Olyan kmiai mreg, amely az egszsgre nagyon rtalmas, a teljes szervezetet, fknt a kzponti idegrendszert krostja. A szemly kls s bels leplshez, szemlyisgnek elnyomorodshoz s hallhoz vezet. A szervezetbe kerlve eufrit, sznes lmokat, csapong gondolatokat, hallucincit, rzkcsaldsokat, ltsi zavarokat, koncentrcis zavarokat, szv s rproblmkat, agylgyulst s lgzsi zavarokat idz el. minsts: Az 1960as vekben a devins viselkedsnek egy jfajta szociolgiai elmlete jelent meg, az n. minsts vagy cmkzsi (labelling) elmlet (Becker 1963, Lemert 1967). Ezen elmlet szerint nem magn viselkedsen, hanem a trsadalomnak, valamint egyes erre kijellt trsadalmi intzmnyeknek (rendrsg, brsg, pszichitriai intzmnyek) vlaszreakcijn mlik, hogy valamilyen viselkeds vagy szemly devinsnak minsl e. viktimolgiai felvtel: Nmely esetben gy prbljk a bnzs elterjedst felderteni, hogy a lakossg reprezentatv mintjt krdezik meg, hogy kvetteke elellene egy meghatrozott idn bell (pldul mlt vben) bncselekmnyt vagy valamilyen meghatrozott fajta bncselekmnyt. Ezeket nevezik vitkimolgiai felvtelnek. letprevencia rtk A drogfogyaszts letprevalenciartke azok arnyt mutatja, akik letk sorn fogyasztottak mr valaha valamilyen drogot. Ezt rtelmezhetjk a tiltott s leglis drogok tartomnyra, vagy valamilyen konkrt szerre vonatkozan, ennek megfelelen hasznljuk a tiltott illetve a leglis szerek letprevalalencia rtke, valamint pl. a marihuna letprevalencija kifejezseket. Ez a legltalnosabb mutat. Emellett megklnbztetnk ves s havi prevalencia rtkeket is, ami mr az aktulisabb elz vi, illetve a krdezst megelz hnapra vonatkoz fogyasztsi arnyokat jelzi.

Vitakrdsek 1. Milyen szerepe lehet a magyar kultra sajtossgainak az ngyilkossg, az alkoholizmus s bizonyos mentlis zavarok gyakorisgban Magyarorszgon? A magyarorszgi empirikus devianciakutatsok kimutattk, hogy az alkoholistk szlei kztt, s mg inkbb a bartimunkatrsi krnyezetben gyakran ittak. Vannak adatok az ngyilkossgok csaldi halmozdsrl is. A legismertebb plda: Teleki Lszl, Teleki Pl s Teleki Gza: mindhrman ngyilkossg kvetkeztben hunytak el. Teleki Pl utalt is tvolabbi rokonnak, Teleki Lszlnak ngyilkossgra, mint bizonyos esetekben kvetend plda. Hasonl pldamechanizmusok minden valsznsg szerint rvnyeslnek sok tlagos helyzet magyar csaldban is. Elg arra utalnunk, hogy a XIX. s XX. szzadban hny, mindenki ltal nagyra becslt politikus s alkot mvsz vetett vget letnek (Szchenyi Istvn, Jzsef Attila, stb.). gy az ngyilkossghoz tbbkevsb kimondottan a hsiessg kpzete kapcsoldik. Az nkntes hall azonban a munkssg s parasztsg kultrjban a krnyezetben megismert pldk hatsra szintn elfogadhat bizonyos esetekben, st taln kvetend megoldsknt szerepel. Felttelezhetjk teht, hogy a meghvott hall a magyar trsadalom kultrjban a szorongstl val menekls lehetsges tjaknt jelenik meg. Az ivs mg inkbb elfogadott, st inkbb ktelez viselkeds a trsadalomban. Az iskolkban elsajttott kultra is elkerlhetetlenl kzvetti. Pl.: a Bordalt els szm opernkban, a Bnk bnban, vagy az Egri csillagok filmvltozata (Dob Istvn lerszegedse). Szinte teljesen elterjedt a magyar trsadalomban az a norma is, hogy az let nagy fordulpontjai, mint az rettsgi, diplomaoszt, bevonuls s leszerels a seregbl, mrtktelen ivssal illik nnepelni. Inkbb csak hipotzissel lehet azt megfogalmazni, hogy a magyar kultrban elterjedt viselkedsi minta a ktsgbeess, a helyzet remnytelensgnek hangslyozsa, a problmk megoldsrl val lemonds. 2. Milyen szerepe lehet a szocialista rendszer sajtossgainak az emltett devins viselkedsek gyakorisgnak nvekedsben Magyarorszgon? A volt szocialista orszgokban nagy s emelked a nem regisztrlt alkoholfogyaszts mrtke, ezrt az ottani viszonyokat nehz megtlni. Az sszehasonltsok alapjn mgis azt felttelezhetjk, hogy a trsgben Horvtorszg s Szlovnia kivtelvel megnvekedett a tmny szesz fogyasztsnak arnya, s itt inkbb jellemzek az alkoholfogyaszts kockzatosabb formi, gy a gyakori nagyivs s lerszegeds. Az egy fre jut sszes szeszes italfogyaszts a legmagasabb rtket 1980ban s 1984ben rte el, ezt kveten cskkent, illetve a kilencvenes vtizedben vltozatlan maradt. Az ezredfordul utn jabb jelents nvekeds mutatkozik. sszefoglalva: az 1990es vekben az alkoholizmus korbban Magyarorszgon jellemz terjedse megllt. A szocialista korszakban a viszonylag kis bnzsi mutatkkal rendelkez orszgok kz tartoztunk, 1990 ta a bncselekmnyek, elssorban a vagyon elleni cselekmnyek szma ntt. 3. Van e mai magyar trsadalomban anmia s elidegenedsvlsg, s ha igen, mi annak az oka? Az anmia s elidegeneds szintjnek mrst elszr Hankiss Elemr s munkatrsai tettek ksrletet Magyarorszgon. Ngy krdst, illetve lltst fogalmaztak meg, melyek bekerltek a Magyar Hztarts Panel 1994. vi felvtelnek krdvbe: azt az rzst, hogy a megkrdezett szemly rtelmetlennek, cltalannak rzi sajt lett az nrtkels elvesztst az rtkek s normk megrendlst a jv perspektvahinyt Mind a ngy dimenziban nagymrtkben ntt 1978tl 1990ig azoknak a vlaszoknak a szma, amelyeket az anmia s az elidegeneds tneteinek, megnyilvnulsainak tekinthetnk. Anlkl, hogy az anmia s elidegeneds elmleti krdseiben a vgs szt ki akarnnk mondani, azt megllapthatjuk, hogy felttelezhet, hogy a magyar trsadalomban az elmlt vtizedekben, legalbb 1941 ta, de taln mr 1914 ta slyos anmia s elidegenedsvlsg alakult ki. A vlsg okai lehetnek azok a trtnelmitrsadalmi megrzkdtatsok, a hbors veresgek, az erszakos, legtbbszr kls hatalmak beavatkozsnak hatsra vgbement rendszervltozsok, amelyek 1918 ta a magyar trsadalmat rtk. Szerepet jtszhat azonban mg az 1945 ta vgbement gyors, st erteljes iparosods, a nagy bels vndorls is, amely bomlasztotta a rgi kiskzssgek integrcis erejt. Az anmia fogalmt mellzve egyszeren azt is mondhatjuk, hogy ezek a megrzkdtatsok s gyors vltoztatsok gyakoribb tettek bizonyos feszltsghelyzeteket, pl.: csaldi s letplyakonfliktusokat, amelyeket a devins viselkedsek htterben kimutattak. 4.Kell-e klnbz devins viselkedsnek gyakorisgnak cskkentse rdekben valamit tenni , s ha igen, mit lehet? Minden trsadalomnak egytt kell lnie bizonyos mennyisg deviancival, mert a teljes konformitsra val trekvs minden bizonnyal elfojtja az jtsra val kszsget, s ezltal gtolja a fejldst. Ezt azrt szksges hangslyozni, mert politikusok s llampolgrok egy rsze is hajlamos arra, hogy a devins viselkedsek tnyvel szembekerlve a szigor elnyomst kvnja alkalmazni. Jellemz ebbl a szempontbl, hogy az Amerikai Egyeslt llamokban nem trltk el a hallbntetst. A hallbntets bnzstl elrettent hatst sok klfldi vizsglat cfolta, vagy legalbbis nem igazolta, teht a slyos bnzs prevencijaknt vgrehajtott kivgzsek nem indokolhatak. A kisebb slyossg bncselekmnyek elkvetstl is inkbb a lebuks kockzata, mint a vrhat bntets a visszatart.

5. Milyen tnyezk jtszhatnak szerepet a szzezer fre jut, ngyilkossgok orszgok szerinti rangsornak megvltoztatsban, az eltr tendencik alakulsban? Noha az eddigi elemzsek nem tudtak tfog, minden orszgra rvnyes s minden szempontbl kielgt magyarzatot adni az ngyilkossgi arnyokban elllt, trtneti lptkben is pldtlan fordulatra, rszsszefggseket sikerlt feltrniuk. Az sszes volt szocialista orszgot s szovjet tagkztrsasgot vizsgl Makinen szerint a legrzkenyebb indiktornak a trsadalmi szinten mrt stressz, a trsadalmi dezintegrltsg s az alkoholfogyasztst mrtknek vltozsa tekinthet, br a legutbbirl felttelezhet, hogy kzvett szerepet jtszik. A demokratizlds kvetkezmnyeit s a gazdasgi helyzet alakulst ler vltozk hatsa nem bizonyult szignifiknsnak. Ugyanakkor azt is hangslyozza, hogy az egyes orszgcsoportokra, illetve orszgokra vonatkoz magyarz smk eltrnek, ami mind az ngyilkossgok szintjt, mind pedig a vltozsok irnyt s mrtkt tekintve orszgspecifikus, kulturlis tnyezk jelentsgre hvja fel a figyelmet. Moksony magyarorszgi vizsglatai is a kulturlis faktorok jelentsgre figyelmeztet. A dlkelet magyarorszgi szletsi hely, az ott szocializlt kulturlis mintk szignifiknsan nvelik az ngyilkossgi kockzatokat (Moksoly 2003). A kedvez hazai tendencikat illeten azt kell mg megjegyeznnk, hogy ezt tbben is elssorban a korbbi frusztrlt helyzeteket felold demokratizldssal magyarzzk, de felvetdik a depresszi hatkonyabb gygyszeres kezelsnek pozitv hatsa is. 6. Mi jellemzi a magyarorszgi kbtszer fogyasztst? Milyen ismrvek jellemzik a leginkbb veszlyeztetett csoportokat? A kbtszerfogyaszts az utols vtizedekben megjelent a magyar trsadalomban, de jelenleg mg nem lehet arrl beszlni, hogy jrvnyszeren terjedne. Ennek gtat vet egyebek kztt az is, hogy Magyarorszgon nagyon szk a fizetkpes kereslet a nemzetkzi kbtszerkereskedelemben szerepet jtsz kemny drogok irnt. Magyarorszg 1995tl kapcsoldott be a fiatalokra koncentrld eurpai ESPADvizsglatokba (European School Survey Project on Alcohol and other Drugs), s azta vannak rszletes adataink a fiatalok kbtszer s alkoholfogyasztsrl. Ezen felmrsek alapjn megllapthatjuk, hogy Magyarorszgon a tiltott szerek fogyasztsa a kilencvenes vtized msodik felben kezdett gyors nvekedsbe. Terjedse az ezredfordul utn is folytatdott: az letprevenciartk 2003ban mr 30% magasabb, mint 1999ben volt. Valamennyi rendszervlt orszgra elmondhat, hogy a kbtszerfogyaszts a fiatalok krben terjedben van. Ugyanakkor az elmlt vtizedben bekvetkezett jelents nvekeds ellenre Magyarorszg Eurpa azon orszgai kz tartozik, ahol ma is alacsony a fogyasztsi arny.

You might also like